Sie sind auf Seite 1von 200

.

A 49-year-old man comes to the physician because of increasingly severe pain in his left hip during the past
2 months. He was recently discharged from a rehabilitation facility for treatment of alcoholism. He says , "The pain
gets so bad at times that it even hurts my golf game. " He has pain with weight bearing. Physical examination shows
limited active and passive range of motion of the left hip . Hip x-rays show no abnormalities A coronal Trweighted
MRI is shown. Which of the following is the most likely cause of this patient's pain?
A) Avascular necrosis
B) Hyperparathyroidism
C) Osteitis deformans
D) Septic arthritis
E) Tuberculous osteomyelitis
A 16-year -old girl has pain and tingling in her left hand when she goes backpacking. The symptoms resolve 5 minutes after she takes off her backpack. Which of the following
abnormalities is the most likely cause?

A) C-7 to T-1 congenital fusion


B) Cervical rib
C) Diastematomyelia
D) Hemivertebra
E) Spina bifida occulta of C-7
A 48-year -old man comes to the physician because of fatigue, increasing shortness of breath on exertion, and decreased exercise tolerance over the past 4 months . An x-ray of the
chest shows cardiac enlargement and prominent pulmonary vasculature. He is treated pharmacologically but subsequently develops glucose intolerance, evidence of liver
dysfunction , and mild to moderate ascites. An older brother died at age 52 years with similar symptoms. Which of the following are the most important screening tests to perform on
this patient and his family?
A) Fasting serum gastrin and insulin measurements
B) Hepatitis B and hepatitis C antibody assays
C) HLA class I and class II typing
D) Serum thyroid-stimulating hormone and thyroxine (T measurements
E) ^
Transferrin saturation and serum ferritin measurements
A 32 - year - old woman begins to hyperventilate and then develops blurred vision and numbness around her mouth after being told her best friend has suddenly died . Her pulse is
95/min and blood pressure is 120/80 mm Hg. Which of the following is most likely decreased in this woman?
A) Arterial blood oxygen concentration
B) Arterial blood Po 2
C) Arterial pressure
D) Cerebral blood flow
E) Cerebral tissue pH
A 2-month-old boy is brought to the physician because of a 7-day history of hoarseness His mother has AIDS and used cocaine throughout her pregnancy. Physical examination
shows no abnormalities A fiberoptic laryngoscopic examination shows a nodule on the left vocal cord. Which of the following viruses is the most likely cause of this nodule?
A) Cytomegalovirus
B) Herpes simplex virus
C) HIV
D) Human papillomavirus
E) Varicella-zoster virus
A 16-year-old girl is brought to the physician because of excessive dieting, constant studying, and social withdrawal during the past 3 weeks. She is 168 cm (5 ft 6 in) tall and
weighs 48 kg (106 lb); BMI is 17 kg/m 2. Her blood pressure is 95/66 mm Hg Physical examination shows dry skin and fine hair growth. Laboratory studies show :
Serum
Na * 138 mEq/L
K+ 2.2 mEq/L
Cl- 90 mEq/L
HCO 3- 36 mEq/L
Urea nitrogen 25 mg/dL
Creatinine 1.2 mg/dL
Arterial blood gas analysis on room air
pH 7.48
Pco 2 48 mm Hg
Po, 97 mm Hg

This patient has most likely been abusing which of the following substances?
A) Alcohol
B) Amphetamines
C) Diuretics
D) Laxatives
E) Thyroid hormone
A 67-year - old woman comes to the physician because of a 1-year history of a nonproductive cough and progressive shortness of breath with exertion. She has not had fever or any
other symptoms. She has no history of serious illness and takes no medications. She does not smoke. Vital signs are within normal limits. Pulse oximetry on room air shows an
oxygen saturation of 92%. Fine, dry crackles are heard at both lung bases. Physical examination shows clubbing of the hands. There is no peripheral edema. A chest x-ray shows
fine linear opacities in the lower third of the lungs bilaterally. A CT scan of the chest shows bibasilar interstitial reticulonodular markings with interlobular septal thickening
Pulmonary function tests show a normal FEVt an FVC of 48% of predicted, and diffusion capacity of the lung for carbon monoxide of 55% of predicted . Which of the following is the
most likely diagnosis?

A) Allergic granulomatous angiitis (Churg-Strauss syndrome)


B) Bronchoalveolar carcinoma
C) Emphysema
D) Idiopathic pulmonary fibrosis
E) Lupus pneumonitis
A 17-year - old boy is brought to the emergency department 45 minutes after he collapsed while competing in a high school wrestling competition He is 178 cm (5 ft 10 in) tall and
normally weighs 61 kg (135 lb), but he competes in the 58-kg (128-lb) weight class. On arrival, he is alert. His pulse is 104/min, and blood pressure is 90/55 mm Hg Physical
examination shows pallor and diaphoresis. After administration of intravenous fluids, he says he feels "back to normal.” On further questioning , he says that he was afraid that he
would not qualify to compete in his weight class, so he took a long steam bath and used a laxative prior to the match to quickly lose 2.3 kg (5 lb). Laboratory studies are most likely
to show which of the following?

A) Hypercalcemia
B) Hyperuricemia
C) Hypokalemia
D) Hyponatremia
E) Hypophosphatemia
A 12-year - old girl is referred to the emergency department by her physician because she has been unable to walk for 5 days. Her parents appear very anxious , but she seems
unconcerned about her symptoms . Her temperature is 37 0°C (98.6°F), pulse is 70/min, and blood pressure is 115/70 mm Hg . Deep tendon reflexes in the lower extremities are
equal bilaterally. Sensation to pain and to light touch is variable. Which of the following is the most likely diagnosis?

A) Brief psychotic disorder


B) Conversion disorder
C) Depersonalization/derealization disorder
D) Illness anxiety disorder (hypochondriasis)
E) Post-traumatic stress disorder
An 85-year -old woman is diagnosed with a fracture of the right femur and begins treatment with morphine by patient-controlled analgesic pump Three days later, her respirations
are 6/min. Physical examination shows pinpoint pupils. Her serum creatinine concentration is 1.8 mg/dL. Which of the following best explains this patient's clinical deterioration after
the initiation of morphine therapy?

A) Chronic dosing of morphine decreases its bioavailability


B) Morphine bioavailability increases with chronic dosing
C) Morphine downregulates p-opioid receptors with chronic dosing
D) Morphine inhibits its own metabolism
E) Morphine is metabolized to active metabolites that accumulate
A 50-year - old man who is a college professor has had an increasing cough for 6 months and hemoptysis for 1 week He has smoked 1 pack of cigarettes per day for the past
32 years. He plays squash twice a week and swims 3 times a week. An x-ray of the chest shows a 3 * 4 -cm hilar mass . Cytologic examination of sputum shows non-small cell
carcinoma . The physician tells the patient he has lung cancer. The patient responds, "How can this be happening to me? I eat right and exercise.” Which of the following responses
by the physician is most appropriate?

A) "Don't worry. I will be there for you and make sure everything turns out the way you want."
B) "Eating right and exercising will not help prevent cancer."
C) "In fact , as you probably know, the major risk for lung cancer is smoking . "
D) "It must be difficult for you to accept this diagnosis when you feel healthy."
E) "Regardless of your good habits, it's time to realize that you have lung cancer."
B, C, D , and
^tfn VAensrtiv
^^ ^^ ^
f0 ® n
itMtVan^ oTeach cutoflpol'nt ''
^^
EX the
cancer? ^
specif ity re p otted gra

100 - 1

1
>
B
c D E

</>
C
A
<D
CO
0
0 1-specificity - 100

O A) OB) O C) O D) O E)
A 49-year-old man has had the gradual onset of numbness and pain of the left index finger over the past 3 weeks Pulses are full and equal bilaterally The left index finger is
mottled blue and pale. Laboratory studies show :
Hemoglobin 14.5 g/dL
Hematocrit 41.7%
Erythrocyte count 4.6 million/mm 3
Leukocyte count 8500/ mm 3
Neutrophils 74%
Eosinophils 4%
Lymphocytes 20%
Monocytes 2%
Mean corpuscular volume 89 pm 3
Platelet count 1, 530,000/mm 3

Examination of bone marrow shows normal cellularity, normal iron stores, and megakaryocytic hyperplasia Which of the following is the most likely diagnosis?

A) Chronic myelogenous leukemia


B) Essential thrombocythemia
C) Myeloid metaplasia
D) Polycythemia vera
E) Reactive thrombocytosis
A study is conducted to assess the extent of cardiac valvular abnormalities in patients receiving ergot dopamine agonists . Group X consists of 40 patients who have been treated
with an ergot dopamine agonist for at least 1 year Group Y consists of 54 age- and gender -matched controls It is found that patients in Group X have significantly greater
composite regurgitation scores (5 ± 2.1 [mean ± standard deviation {SD}]) than Group Y (3.3 ± 2 [mean ± SD]), p < 0.05 . The investigators decide to double the number of patients
in each group, and keep the same criterion (p < 0.05) for statistical significance. This change will most likely have which of the following effects on probability of Type I and Type II
error?
Type I Error Type II Error
A) Increased increased
B) Increased no change
C) Increased decreased
D) No change increased
E) No change no change
F) No change decreased
G) Decreased increased
H) Decreased no change
I) Decreased decreased
A previously healthy 26-year-old woman comes to the physician because of a 2 -week history of fever, chills, malaise, rash, diarrhea, headache, and sore throat . Physical
examination shows an erythematous throat , diffuse swelling of the lymph nodes, and an erythematous , maculopapular rash over the trunk and extremities. Laboratory studies show
a leukocyte count of 10,000/mm3, CD4+ T-lymphocyte count of 700/mm3 (N>500), and plasma HIV viral load greater than 1 million copies/mL. Results of a serum HIV antibody test
are negative. Which of the following best explains this patient's condition?
A) Acute retroviral infection
B) HIV-2 infection
C) HTLV infection
D) Late-stage AIDS
A 50-year - old man is admitted to the hospital because of severe chest pain for 30 minutes. An ECG shows ST-segment elevation in the anterior chest leads. Serum studies show
increased creatine kinase activity and troponin I concentration. Compared with the electrolyte content of normal myocardiocytes found in a healthy, asymptomatic 50-year-old man
which of the following best reflects the electrolyte content of the myocardiocytes that produced the increased enzymes in this patient?
Intracellular [Na +] Intracellular [K +] Intracellular [Ca 2+]
A) Decreased decreased decreased
B) Decreased decreased increased
C) Decreased increased increased
D) Increased decreased decreased
E) Increased decreased increased
F) Increased increased increased
A 43- year - old man is brought to the emergency department 1 hour after forcible eversion of his left foot while jogging on uneven ground. Walking is limited by pain. He rates the
pain as 7 on a 10-point scale His vital signs are within normal limits Physical examination shows tenderness and swelling over the lateral malleolus . An x-ray of the left lower
extremity shows a fracture of the fibula above the distal tibiofibular articulation Contraction of which of the following muscles most likely caused this patient's pain at the fracture?

A) Extensor digitorum longus


B) Fibularis (peroneus) brevis
C) Fibularis (peroneus) tertius
D) Flexor digitorum longus
E) Tibialis anterior
F) Tibialis posterior
A 28-year - old man who is seropositive for HIV has numerous nonulcerated purple nodules on the skin. Examination of tissue obtained on biopsy of a lesion is most likely to show
which of the following?
A) Aggregates of large atypical lymphocytes
B) Granulation tissue containing pseudohyphae and budding yeasts
C) Intracellular yeasts in macrophages
D) Macrophages containing acid-fast bacilli
E) Multinuclear cells containing intranuclear inclusions
F) Slit-like vascular spaces with plump spindle-shaped stromal cells
A 73-year-old man comes to the physician with his wife because of a 3-year history of daytime sleepiness His wife says that he snores loudly. He is 183 cm (6 ft) tall and weighs
113 kg (250 lb); BMI is 34 kg/m 2. His pulse is 84/min , respirations are 18/min, and blood pressure is 175/105 mm Hg. Physical examination shows congested conjunctivae and
centripetal obesity. Serum studies show a glucose concentration of 250 mg/dL, creatinine concentration of 2 mg/dL, and uric acid concentration of 10.1 mg/dL Polysomnography
confirms the diagnosis, and treatment with nasal continuous positive airway pressure at night is started. This therapy will most likely cause a decrease in which of the following?
A) Blood pressure
B) Pulse rate
C) Serum creatinine concentration
D) Serum glucose concentration
E) Serum uric acid concentration
A 30-year-old man and a 24-year-old woman (indicated by the arrows) request preconceptional counseling because the man's brother has
type I oculocutaneous albinism. This is an autosomal recessive, single gene disorder thought to occur about once in every 40 , 000 persons in
the general population Assume that the alleles are in equilibrium in the population, that the woman' s family history is negative for this disorder,
and that the two probands have neither shared relatives nor common ethnicity A pedigree is shown Which of the following is the best estimate

6
that their child will be affected with oculocutaneous albinism?
i A) 1 in 4
B) 1 in 100
C) 1 in 200 * *
D) 1 in 600
E) 1 in 40, 000
A 65-year - old man starts using topical fluorouracil for treatment of actinic keratoses on his scalp. Fluorouracil is most likely effective for this patient because it inhibits which of the
following enzymes?

A) Thymidine kinase
B) Thymidine reductase
C) Thymidylate synthase
D) Uracil phosphoribosyltransferase
E) Uracil reductase
A 73- year - old man has difficulty urinating and frequent urination. Improvement of these symptoms is most likely to result from treatment with a drug that blocks which of the following
labeled sites?

Tyrosine

*4
DOPA
Choline

4i
» Storage

** granule m Acetylcholine

Bti I c Q Acetylcholinesterase
Reuptake carrier

-® rn —
f Acelylchollne receptor
H

O A) OB) O C) O D) O E) OF) O G) OH) O I)


A 32 - year - old man comes to the physician because of a 2-week history of persistent thrush. He has been receiving a multidrug regimen including zidovudine (AZT) for 3 years, but
he has had difficulty with compliance. Laboratory studies show a CD4+ T-lymphocyte count of 100/mm 3 (N > 500) and plasma HIV viral load of 100,000 copies/mL. HIV genotyping
shows a mutation consistent with zidovudine resistance. This mutation is most likely in which of the following viral genes?
A) DNA polymerase
B) Glycoprotein 41
C) Glycoprotein 120
D) Integrase
E) Protease
F) Reverse transcriptase
G) Thymidine kinase
A 29-year-old woman is brought to the emergency department 1 hour after being injured in a motor vehicle collision. Abdominal examination shows tenderness with rebound . Vital
signs are stable. A CT scan of the abdomen shows no abnormalities except for a small amount of intraperitoneal fluid . After 6 hours of observation, an exploratory laparotomy is
done because of continuing severe abdominal pain No abnormalities are seen until the surgeon gently slides his hand behind the liver; there is a sudden massive hemorrhage from
the region of the hand, and the patient's blood pressure becomes undetectable within 1 minute Which of the following underlying lesions is most likely associated with the
hemorrhage?

A) Avulsion of celiac trunk from the aorta


B) Avulsion of hepatic veins from the inferior vena cava
C) Traumatic transection of the aorta
D) Traumatic transection of the hepatic artery
E) Traumatic transection of the portal vein
A 26-year - old woman develops hypotension and hemoglobinuria after receiving 1 unit of packed red blood cells. Which of the following are most likely to be involved in her
condition?
A) Antibody, complement C5-9
B) Circulating immune complexes, complement C5a, neutrophils
C) Cytotoxic T lymphocytes
D) IgE, mast cells , histamine, cytokines
E) Lymphocytes, cytokines, macrophages
A 74 - year - old man with emphysema and lung cancer is brought to the clinic by a social worker because of his wasted appearance. His wife of 50 years died 8 months ago He is
impoverished, and his diet consists primarily of cereal and toast. He has lost 13.6 kg (30 lb) in the past 6 months. He has temporal muscle wasting , sunken eyes , loose skin , and
edema of the lower extremities. He appears despondent and withdrawn. Serum albumin concentration is 2.5 g/dL. An x-ray of the chest shows a large mass in the hilum of the right
lung . Which of the following best explains the edema?

A) Anabolic state
B) Glycogen depletion
C) Lipolysis
D) Negative nitrogen balance
E) Starvation ketosis
A 22-year - old woman who is at 16 weeks' gestation has decreased serum concentrations of a-fetoprotein and unconjugated estriol and an increased serum concentration of human
chorionic gonadotropin . Which of the following disorders affecting the fetus is most likely?
A) Angelman syndrome
B) Beckwith-Wiedemann syndrome
C) Down syndrome
D) Fragile X syndrome
E) Myotonic dystrophy
F) Prader-Willi syndrome
A 50-year - old man comes to the physician 3 days after his first generalized tonic-clonic seizure. He has a 1-month history of frequent episodes of a pins-and-needles sensation
around the mouth and in the hands and feet. He also has had occasional episodes of involuntary contraction of the muscles of the hands or feet leading to wrist flexion or foot
plantar flexion. His pulse is 80/min, and blood pressure is 150/ 90 mm Hg Neurologic examination shows mild, diffuse hyperreflexia. An abnormality of which of the following serum
electrolyte concentrations is the most likely explanation for his symptoms?

A) Bicarbonate
B) Calcium
C) Chloride
D) Potassium
E) Sodium
A 50-year - old man who has smoked 2 packs of cigarettes a day for 34 years has experienced gradually increasing shortness of breath on exertion , chronic cough productive of
thick sputum, and frequent episodes of wheezing The anteroposterior diameter of his chest is increased Diminished breath sounds and scattered rhonchi are audible throughout
the lung fields. Which of the following laboratory abnormalities is expected?

A) Decreased blood Pco 2


B) Increased blood HC03-
C) Increased blood pH
D) Increased urinary excretion of HC03-
E) Increased urinary pH
I
severe ankle injury. Physical examination shows weak extension or m<
nerve injury at which of the following labeled locations in the diagram?
An unimmunized 1-year -old boy is admitted to the hospital because of fever, irritability, and a stiff neck. Cultures of the nasopharynx and cerebrospinal fluid grow gram-negative
coccobacilli. Electron microscopic examination shows the presence of pili on the bacteria recovered from the nasopharynx, but absence of pili on the bacteria of the same species
recovered from the cerebrospinal fluid Which of the following processes shuts off expression of pili in this bacterium following invasion?
A) Horizontal gene transfer
B) Phase variation
C) Plasmid acquisition
D) Point mutation
E) Transposon mutagenesis
A previously healthy 7-year -old girl is brought to the physician by her parents because of a 1 -month history of excessive urination; she also has had a 2.3-kg (5-lb) weight loss
during this period. Her pulse is 110/min , and blood pressure is 92/58 mm Hg. Physical examination shows poor skin turgor and a fruity odor on her breath. Her blood glucose
concentration is 612 mg/dL. Laboratory studies are most likely to show an increase in which of the following?
A) Arterial pH
B) Serum bicarbonate concentration
C) Serum chloride concentration
D) Serum potassium concentration
E) Serum sodium concentration
A previously healthy 12-year -old boy is brought to the emergency department because of a 6-hour history of severe diarrhea. His temperature is 37.0° C (98.6°F), pulse is 100/min,
respirations are 16/min, and blood pressure is 110/70 mm Hg. He appears markedly dehydrated . Physical examination shows no other abnormalities . The most appropriate
treatment for this patient is an oral rehydration solution containing which of the following sets of ingredients?

A) Bicarbonate and potassium


B) Bicarbonate and sodium
C) Glucose and insulin
D) Glucose and sodium
E) Lactate and bicarbonate
F) Lactate and potassium
A 4-month-old boy is brought to the office by his parents because of his small size, and difficulty feeding from a bottle. His mother states that
he was recently adopted from a developing country, and his medical history is unknown . His vital signs are within normal limits. He is at the
10th percentile for weight and 40th percentile for length. Physical examination shows the findings in the photograph. The most likely cause
of the facial finding involving the lip in this patient is a failure of normal fusion of which of the following pairs of structures?
A) Lateral nasal and mandibular prominences
B) Lateral nasal and maxillary prominences
C) Lateral nasal and medial nasal prominences
D) Mandibular and maxillary prominences
E) Mandibular and medial nasal prominences
F) Maxillary and medial nasal prominences
A 24-year - old woman, gravida 1, para 1, comes to the physician because of nervousness and tremor since delivering a healthy female newborn 6 weeks ago. She also has had a 5-
kg ( 11-lb) weight loss during this period . She has a history of panic disorder and carpal tunnel syndrome. Her temperature is 37°C ( 98.6°F), pulse is 100/min, and blood pressure is
,
140/80 mm Hg . Physical examination shows a firm thyroid gland that is twice the normal size. Serum studies show a free thyroxine (FT, ) concentration of 2.4 ng/dL (N=0.8-2.4),
thyroid-stimulating hormone concentration of less than 0.03 pU/ mL, and 24-hour thyroid radioactive iodine uptake of 1% (N=8-25%) . Which of the following is the most likely
explanation for this patient's symptoms?

A) Anaplastic thyroid carcinoma


B) Hypothyroidism
C) Release of stored thyroid hormone from a thyroid gland infiltrated by lymphocytes
D) Release of thyroid hormone from a lymphomatous thyroid gland
E) Release of thyroid hormone from a thyroid gland stimulated by antibodies
!. A 10-year - old girl has a slightly painful 2-mm subcutaneous nodule on her chin. Ten weeks ago, she required sutures after she | •
4
lacerated her chin playing basketball. A photomicrograph of tissue obtained on biopsy of the nodule is shown. Which of the ' . *
n. *> *
• • • 8
following best describes the pathologic features? / m V
i , >; % 1 vI ,
A)
B)
Amyloid
Granulation tissue t
/

C) Granuloma
) D) Lymphocytic infiltrate
E) Neutrophilic infiltrate
A 5-year-old boy with mental retardation is grossly obese and has facial features of Prader-Willi syndrome (PWS). Karyotyping and fluorescent in situ hybridization studies do not
show deletion in the usual site on chromosome 15 . Which of the following findings is most likely to confirm PWS in this child?

A) Deletion in the short arm of chromosome 15


B) Duplication within chromosome 15
C) Large trinucleotide repeat expansion in the PWS area of chromosome 15
D) Maternal origin of both chromosomes 15
E) Translocation in the short arm of chromosome 15
A 45-year- old man who is HIV positive comes to the physician because of a 1-week history of headaches and blurred vision. A lumbar puncture is done. Laboratory studies show:
Serum glucose 95 mg/dL
Cerebrospinal fluid
Appearance cloudy, straw-colored
Leukocyte count 400/mm 3 (100% lymphocytes)
Erythrocyte count 50/mm 3
Glucose concentration 30 mg/dL
Total protein concentration 68 mg/dL
Fungal stain positive for yeast

The most appropriate treatment for this patient is a drug with which of the following mechanisms of action?
A) Antagonism of murein hydrolase
B) Blockade of microtubule polymerization
C) Disruption of the pathogen cell membrane
D) Induction of cytochrome P450 enzymes
E) Inhibition of pathogen cell wall synthesis
In patients with adenosine deaminase deficiency, there is a 50- to 100- fold increase in dATP concentration in T lymphocytes. This increase in dATP inhibits which of the following
enzymes , thus compromising DNA synthesis?
A) DNA ligase
B) DNA polymerase I
C) Nucleoside monophosphate kinase
D) Ribonucleotide reductase
E) Thymidylate synthase
Patients with mucolipidosis II (l-cell disease) lack the phosphotransferase required for the formation of mannose 6-phosphate residues that are normally attached to enzymes
destined for the lysosome. The mannose 6-phosphate sorting signal is normally added to proteins in the c/s-Golgi. Which of the following most likely occurs to the lysosomal
enzymes in these patients?

A) Degraded in the lysosome


B) Imported into the nucleus
C) Retained in the rough endoplasmic reticulum
D) Secreted from the cells
E) Sorted into the mitochondria
A 37-year - old man comes to the physician for a follow -up examination. He has a 1-year history of hypertension. Despite aggressive pharmacotherapy, his blood pressure has
averaged 188/131 mm Hg. His blood pressure today is 190/135 mm Hg Funduscopic examination shows multiple flame hemorrhages with severe arteriolar narrowing. Which of the
following disorders of blood vessels is most likely to be caused by this condition?

A) Berry aneurysm
B) Fusiform aneurysm
C) Hyperplastic arteriolosclerosis
D) Medial arterial calcification (Monckeberg arteriosclerosis )
E) Temporal arteritis
A case-control study is conducted to assess the relationship between exposure to environmental tobacco smoke and sinusitis. The table shows the results of the study
Cases Controls
Exposed to smoke 20 25
Not exposed to smoke 5 100

Which of the following is the prevalence of sinusitis in the general population?


A) 16%
B) 40%
C) 50%
D) 80%
E) Cannot be determined from the data given
A 52-year - old man with metastatic oat cell carcinoma begins to receive hospice care. Treatment with narcotic medication is initiated for severe bone pain. Which of the following
actions by the physician regarding this treatment is most appropriate?
A) Begin with minimal amounts of pain medication to monitor any adverse effects
B) Carefully monitor the administration of this pain medication to avoid patient addiction
C) Ensure that the patient receives enough medication to control his pain
D) Make sure that the patient does not have direct access to the pain medication to avoid intentional overdose
E) Regularly assess the patient for respiratory depression due to the pain medication
A healthy 25-year - old man eats a meal consisting of 60% carbohydrates, 30% fat, and 10% protein. His serum insulin concentration is increased after the meal . Which of the
following is the most likely mechanism of pancreatic secretion of insulin in this man?

A) Diffusion of insulin across the plasma membrane by an increase in intracellular bicarbonate concentration
B) Diffusion of insulin across the plasma membrane through a glucose transporter
C) Diffusion of insulin across the plasma membrane through a protein transporter
D) Enzymatic hydrolysis of the plasma membrane
E) Fusion of an intracellular vesicle with the plasma membrane
i. A 60-year -old man comes to the physician because of progressive shortness of breath during the past 3 months. He
states that for many years he has worked in a shipyard. His respirations are 25/min . Bilateral basilar crepitant
crackles are heard. An x-ray of the chest shows reticulonodular pulmonary infiltrates consistent with interstitial
fibrosis. Microscopic examination of sputum shows occasional elongate structures as shown. The fibrosis was most
likely initiated by interaction of these structures with which of the following cell types?
A) Alveolar capillary endothelial cell O F) Goblet cell
B) Alveolar macrophage G G) Kulchitsky cell
C) Chondrocyte O H) Squamous epithelial cell
D) Ciliated columnar epithelial cell O I) Type I pneumocyte
E) Clara cell G J) Type II pneumocyte
A 65-year - old woman with a 20- year history of osteoarthritis of the hands now has pain radiating down the distal anterior thigh, knee, medial leg , and foot. Bony outgrowth of
vertebrae compressing one of the spinal nerves is suspected. Compression of a nerve root in which of the following intervertebral foramina is the most likely cause of her
symptoms?

A) T-10 to 11
B) L-1 to 2
C) L-3 to 4
D) S-1 to 2
E) S-4 to 5
-
A 15 year- old girl is brought to the physician by her mother because of a 1-day history of redness and painful skin following sunbathing. She reports that she used sunblock every
few hours. She takes no medications Physical examination shows severe erythema of the back and extremities. There are no blisters Which of the following best describes her
condition?
A) Allergic reaction to the sun
B) First-degree burn
C) Rhus dermatitis
D) Second-degree burn
E) Third-degree burn
A 48-year-old man begins furosemide therapy for pedal edema associated with biventricular failure and hypertension Five days later, the edema is not fully resolved, and his serum
potassium concentration has decreased from 4.2 mEq/L to 3 mEq/L. A drug with which of the following actions should be added to this patient's medication regimen?

A) Blocks basolateral K + channels in the collecting duct


B) Decreases the luminal permeability to Na + in the collecting duct
C) Increases the delivery of Na + and K + to the collecting duct
D) Increases the negative charge of the luminal tubule fluid
E) Stimulates Na +-K + ATPase in the collecting duct
A 10-month-old girl is brought to the emergency department because she is pale and unresponsive . Her mother has given her 3 adult doses of loperamide over the past 6 hours for
diarrhea. Her pulse is 120/min and respirations are 10/min. Examination shows marked abdominal distention. If medication is required , which of the following is the most
appropriate therapy?
A) Bethanechol
B) Diphenhydramine
C) Epinephrine
D) Naloxone
E) Physostigmine
A 28-year -old woman is brought to the physician because of a 3-week history of double vision. One year ago, she had an episode of blurry vision in her left eye that gradually
improved during the next 2 months without treatment Neurologic examination shows that the right eye does not adduct past the midline on horizontal gaze when looking to the
left When convergence is tested , the right eye is able to adduct past the midline. Rightward horizontal gaze is normal. This patient most likely has a lesion involving which of the
following labeled structures in the photograph of the brain stem shown?
A 9-year-old boy is brought to the physician by his mother because of a 1-year history of cough productive of mucoid sputum, wheezing , and shortness of breath with exertion He
has a history of recurrent upper respiratory tract and sinus infections since birth. He is at the 25th percentile for height and weight. The mother says that his younger sibling is
beginning to develop similar problems. Physical examination shows mild clubbing of the fingers Laboratory studies show markedly increased sweat chloride and sodium
concentrations. A defect of which of the following in this patient's bronchial epithelium is most likely causing these symptoms?
A) Adrenoreceptors
B) Membrane receptors
C) Nuclear receptors
D) Protein regulation
E) Protein structure
A 25-year - old woman is admitted to the hospital because of septic shock She underwent splenectomy 6 months ago after she sustained traumatic injury during a motor vehicle
collision . Her temperature is 39 4°C (103'F), and blood pressure is 80/40 mm Hg Coarse tubular breath sounds are heard bilaterally, and egophony is heard at the right base. A
chest x-ray shows bilateral pulmonary opacities, dense consolidation at the base of the right lung , and a possible right- sided effusion . Which of the following is the most likely causal
organism?

A) Actinomyces israelii
B) Bacillus anthracis
C) Klebsiella pneumoniae
D) Pseudomonas aeruginosa
E) Streptococcus pneumoniae
fhrou l
A) 10^ h e r a t e (i n Umin) mrough the aneurysm .
The estimated cross-sectional area of the aneurysm is 2 cm 2 and the mean velocity of blood flow

B) 2.4
C) 3.2
D) 3.6
E) 4.0
A 66-year -old man develops worsening shortness of breath and swelling of the legs. He had a myocardial infarction 6 months ago. Pulse is 120/min, and blood pressure is 135/ 82
mm Hg Examination shows distended jugular veins, bilateral crackles, and pitting edema in the lower extremities. Serum sodium concentration is 129 mEq/L . Which of the following
is most likely responsible for the hyponatremia in this patient?

A) Dilution of serum sodium due to ADH (vasopressin) secretion


B) Increased urinary sodium excretion due to renal tubular dysfunction
C) Increased urinary sodium excretion due to secretion of aldosterone
D) Increased urinary sodium excretion due to secretion of atrial natriuretic peptide
E) Restriction of dietary sodium intake
). A 30-year- old man comes to the physician because of a 2-day history of blood in his urine, dull flank pain, and voiding
large amounts of urine. The patient states that his father died of an unknown kidney disease . His blood pressure is
150/100 mm Hg. Physical examination shows bilateral renal masses. Laboratory studies show proteinuria and gross
hematuria A CT scan of the abdomen shows cystic masses in each of the kidneys; the right mass is larger than the left .
A right nephrectomy is done; the surgical specimen is shown. The underlying defect most likely involves which of the
following molecules?
A) Desmocollin-1a
B) E-cadherin
C) Laminin receptor
D) P-selectin
E) Polycystin
A 60-year - old woman comes to the physician because of a 2-month history of shortness of breath and cough and a 2-week history of progressive swelling of her face. The cough is
productive of approximately V cup of mildly blood-tinged, white-to-yellow sputum daily. She has had a 2.3-kg (5-lb) weight loss over the past 4 months. She has a 15-year history of
* . She has smoked 1 pack of cigarettes daily for 45 years. Her pulse is 90/min and regular, respirations are 20/min , and blood pressure is 135/90
hypertension treated with enalapril
mm Hg . Physical examination shows edema of the face and neck and jugular venous distention. Breath sounds are mildly decreased throughout all lung fields, and there are no
cardiac murmurs . Her serum sodium concentration is 125 mEq/ L. Which of the following is the most likely underlying cause of this patient's facial edema ?

A) Angioedema
B) Hyperthyroidism
C) Hyponatremia
D) Hypoparathyroidism
E) Lung cancer
A 57-year - old woman comes to the physician because of episodes of burning, shock-like pain on the left side of her face during the past 4 months . She has several episodes each
day. usually caused by chewing. The pain lasts from 5 to 10 seconds. Neurologic examination shows no abnormalities . Which of the following is the most likely diagnosis?

A) Cluster headache
B) Migraine
C) Multiple sclerosis
D) Temporal arteritis
E) Trigeminal neuralgia
A 42-year -old man comes to the physician because of a 1-month history of fatigue, excessive thirst , and frequent urination with large volume. He is 178 cm (5 ft 10 in) tall and
weighs 61 kg (135 lb); BMI is 19 kg/ m 2. His pulse is 96/min, and blood pressure is 104/60 mm Hg. Physical examination shows dehydration . Fasting laboratory studies show:
Serum
Glucose normal
ADH (vasopressin) increased
Osmolality increased
Urine osmolality decreased

In addition to a low- sodium diet, which of the following drugs is the most appropriate treatment for this patient?
A) Acetazolamide
B) Desmopressin
C) Furosemide
D) Hydrochlorothiazide
E) Spironolactone
A 1-year- old boy is found to have an impairment of the respiratory burst of phagocytes. He has a family history of immunodeficiency. This patient is at greatest risk for infection
caused by which of the following organisms?
A) Enterococcus faecium
B) Escherichia coli
C) Haemophilus influenzae
D) Staphylococcus aureus
E) Streptococcus pneumoniae
F) Streptococcus pyogenes (group A)
A 13-year - old girl is brought to the physician by her mother for a routine well-child examination. Neither the child nor the mother has any particular health concerns. The patient is
doing well in school and participates in soccer and lacrosse in a local recreation league The physician wants to ask the patient about sexuality and sexual activity. Which of the
following actions by the physician is most appropriate?

A) Ask the mother to leave the room before asking the patient any questions
B) Ask the patient to leave the room and then ask the mother about her daughter
C) Ask the questions with both the mother and the patient present
D) Give the patient a questionnaire that she can fill out at home
E) Tell the patient that some sensitive questions are next and that only she can ask her mother to leave the room
i. A 28-year-old man comes to the emergency department because of a 6-hour history of cramping and severe left midabdominal pain
He had a single episode of vomiting . His temperature is 37°C (98.6°F). Abdominal examination shows a mass in the midabdomen. A
CT scan of the abdomen with contrast is shown ; the abnormality is indicated by the arrows Intussusception of which of the following
portions of the intestinal tract is the most likely cause of this patient’s pain?
A) Appendix
B) Cecum
C) Duodenum
D) Jejunum
E) Stomach
An Rh- negative woman, gravida 3, para 2, has had an increasing bilirubin concentration in amniotic fluid obtained between 22 and 29 weeks' gestation . Umbilical cord blood
obtained at 29 weeks' gestation had a hematocrit of 6.2%. Transfusion of which of the following types of packed red cells to the fetus in utero should be used to correct the anemia'?
A) A, Rh-negative
B) A, Rh-positive
C) AB , Rh-negative
D) AB, Rh-positive
E) B, Rh-negative
F) B, Rh-positive
G) O, Rh-negative
H) O, Rh-positive
A 35-year - old woman has weakness and fever When she was 25 years old, she was treated for Hodgkin disease with mechlorethamine, oncovin, procarbazine, and prednisone.
She has had a splenectomy There is no evidence of lymphadenopathy. A peripheral blood smear shows numerous blast forms. Which of the following is the most likely explanation
for these findings?

A) Acute lymphocytic leukemia


B) Acute myelocytic leukemia
C) Chronic lymphocytic leukemia
D) Lymphoma
E) Recurrence of Hodgkin disease
During an experiment , a 22-year-old man receives an infusion of histamine in his left brachial artery. Which of the following sets of microcirculatory changes will most likely occur in
his left arm?
Arteriolar Resistance Capillary Hydrostatic Pressure Capillary Filtration Rate
A)
B)
C) T i
D) T t
E) i i
F) i T
G) i i
H) i t
A 25 - year -old woman comes to the physician because of easy bruising and bleeding from her gums over the past 2 days . She has been otherwise healthy and has never had
abnormal bleeding. Laboratory studies show:
Hemoglobin 13.2 g /dL
Hematocrit 39.6%
Leukocyte count 7000/mm 3
Platelet count 6000/mm 3
Prothrombin time 14 sec (INR=1.2)
Partial thromboplastin time 30 sec

Which of the following is the most likely cause of these findings?


A) Disseminated intravascular coagulation
B) Hypersplenism
C) Immune thrombocytopenic purpura
D) Iron deficiency
E) Vitamin C deficiency
F) von Willebrand disease
An 11-year-old boy has had pain in his right mid-thigh for 2 weeks. Physical examination shows an area of warmth and tenderness around an ill-defined mass. X-ray of the femur
shows a solitary mid-diaphyseal osteolytic lesion with permeative margins. The lesion is surrounded by concentric layers of reactive bone. Tissue obtained on biopsy of the mass is
composed of poorly differentiated sheets of small, uniform, hyperchromatic cells with a very high nuclearcytoplasmic ratio, scant stroma, and no visible matrix. Which of the
following is the most likely diagnosis?

A) Chondrosarcoma
B) Eosinophilic granuloma
C) Ewing sarcoma
D) Multiple myeloma
E) Nephroblastoma (Wilms tumor)
F) Osteosarcoma
G) Small lymphocytic lymphoma
A 35-year - old man with bronchial asthma starts taking albuterol Which of the following changes in second messenger levels is most likely in the cells affected by this treatment?

A) Decrease in cAMP
B) Decrease in cGMP
C) Decrease in inositol 1, 4, 5-trisphosphate
D) Increase in cAMP
E) Increase in cGMP
F) Increase in inositol 1,4,5-trisphosphate
A 57- year - old woman comes to the physician 2 weeks after finding a lump in her left breast on self -examination Physical examination shows a 2-cm mass in the upper inner
quadrant of the left breast. Carcinoma is suspected. X-rays show multiple lesions in the vertebral column, and metastatic disease is suspected. Which of the following veins draining
the breast provides the most direct pathway for malignant cells to travel to the vertebral column?

A) Axillary
B) Cephalic
C) Intercostal
D) Internal thoracic (mammary)
E) Lateral thoracic
A 10-year - old boy receives a renal transplant from a living , related donor for cystic renal dysplasia. Three months later , there is tenderness at the site of the graft , and serum
creatinine concentration increases from 1.0 to 2.0 mg/dL. Which of the following findings on renal biopsy is most likely to confirm a diagnosis of rejection?
A) Arteriolar C 3 deposition
B) Dilation of Bowman space
C) Fibrous scars with plasma cells
D) Glomerular neutrophils and necrosis
E) Lymphocytes infiltrating tubular epithelium
F) Red blood cell casts
G) Subcortical necrosis
An 18-year-old man comes to the physician 10 days after injuring his right hand during a game of tag football. He says that he was attempting to tag an opposing player when he
inadvertently caught the tip of his ring finger on the other player's jersey. He felt a popping sensation at the tip of his finger and then immediately felt sharp pain localized to the
finger. He noticed swelling of the finger several minutes later. Examination of the right hand shows mild tenderness over the palmar tip of the ring finger. He is unable to flex the
distal interphalangeal joint of the right ring finger, but he is able to flex the metacarpophalangeal and proximal interphalangeal joints. This patient most likely injured a tendon whose
muscle is supplied by fibers from which of the following nerve roots?
A) C3
B) C4
C) C5
D) C6
E) C7
F) C8
A 35 - year - old man comes to the physician because of a 6-week history of pain in his right arm and tingling sensation in the fingers of his right hand Physical examination shows
weakness of extension and pronation of the right forearm. The triceps muscle stretch reflex is decreased Which of the following nervous structures is most likely involved?

A) C6 nerve root
B) C7 nerve root
C) C8 nerve root
D) Median nerve
E) Radial nerve
A 27-year -old woman comes to the physician because of swelling in her left arm that began 10 days after a spontaneous abortion in the middle of her second trimester. A
radiocontrast study shows axillary vein thrombosis. Laboratory studies show:
Bleeding time normal
Prothrombin time increased
Partial thromboplastin time increased
Thrombin time normal

Which of the following is the most likely cause of this woman's condition?
A) Antiphospholipid antibodies
B) Antithrombin deficiency
C) Dysfibrinogenemia
D) Protein C deficiency
E) Protein S deficiency
The risk for hemorrhagic stroke from drug X is investigated by utilizing a questionnaire about drug use in 702 patients with hemorrhagic stroke and in 1376 control subjects
contacted by random-digit dialing. Which of the following best describes this study design?
A) Case-control study
B) Cohort study
C) Cross-sectional survey
D) Randomized, double-blind experiment
E) Randomized, single-blind experiment
. A 4-year-old boy is brought to the physician by his parents because he has had two bacterial urinary tract infections
during the past year. Physical examination shows no abnormalities Radiologic studies of the urinary tract show marked
dilation of the left ureter and renal pelvis, and minimal left-sided renal function. A left nephrectomy is performed. A
photograph of the resected specimen is shown. Microscopic examination of the renal parenchyma is most likely to show
which of the following?
A) Amyloidosis
B) Interstitial inflammation
C) Membranous glomerulonephritis
D) Nephroblastomatosis
E) Renal cell carcinoma
Which of the following is required for the synthesis of glucosamine from fructose 6-phosphate?
A) Arginine
B) ATP
C) Carbamoyl phosphate
D) Glutamine
E) A/-acetylglutamate
A 25- year-old woman comes to the physician because of a 6-month history of irregular menstrual periods and progressive acne. Menses have occurred at irregular 25- to 42-day
intervals. She is 170 cm (5 ft 7 in) tall and weighs 100 kg (220 lb); BMI is 35 kg/m 2. Her voice is deep Physical examination shows dark hair over the upper lip and chin, and open
and closed comedones over the face, chest, and back . Serum studies are most likely to show which of the following in this patient?
Fasting Luteinizing
Insulin Testosterone Hormone
A) Increased increased increased
B) Increased increased decreased
C) Increased decreased increased
D) Decreased increased increased
E) Decreased increased decreased
F) Decreased decreased decreased
A 56- year -old woman is brought to the emergency department by her husband because she has had headache , confusion, and difficulty speaking and walking for the past 2 days
Mental status shows confusion and memory loss Her temperature is 38.9°C (102°F). Brudzinski and Kernig signs are absent. Analysis of cerebrospinal fluid shows:
Opening pressure 260 mm Hp
Leukocyte count
(mostly lymphocytes) 208/mm 3
Protein 52 mg/dL
Glucose 40 mg/dL

An EEG shows abnormalities in the right temporal lobe and periodic lateralized epileptiform discharges. Which of the following is the most likely diagnosis?

A) Aseptic meningitis
B) Bacterial encephalitis
C) Herpes encephalitis
D) Neurosarcoidosis
E) Tuberculous meningitis
A 66-vear - old man comes to the physician because of a 2-month history of the inability to maintain an erection He also has had fatigue and difficulty sleeping and concentrating
during this period. Three months ago , he had a cerebral infarction with right hemiparesis that has gradually resolved. Physical examination shows no abnormalities. Which of the
following additional pairs of findings in this patient is most likely on history taking?

Libido Nocturnal
Erections
A) Normal normal
B) Normal decreased
C) Decreased normal
D) Decreased decreased
A 52-year -old man is brought to the emergency department 1 hour after he was found next to a faulty space heater He is confused . Physical examination shows reddish-tinged skin
Arterial blood gas analysis on room air shows a Po 2 within the reference range and carboxyhemoglobin concentration of 40% (N=2%). He is intubated and mechanically ventilated
with supplemental oxygen. Assuming no further carbon monoxide (CO) exposure, which of the following is the best estimate for the time it will take for the removal of all the
CO-carrying erythrocytes from this patient's circulation?
A) 1 Day
B) 2 Weeks
C) 1 Month
D) 4 Months
E) 1 Year
A randomized controlled trial is conducted to assess the effect of diuretic therapy on the risk for cerebral infarction in patients with systolic hypertension . A total of 4736 patients with
systolic blood pressure measurements greater than 140 mm Hg and diastolic blood pressure measurements ranging from 50 mm Hg to 85 mm Hg are randomly assigned to receive
diuretic therapy or placebo. After 5 years, results show:
Cerebral Infarction
Yes No Total
Diuretic Therapy 123 2242 2365
Placebo 194 2177 2371
Total 317 4419 4736

Which of the following best represents the absolute risk reduction for cerebral infarction in patients receiving diuretic therapy?
A) (194/2177) - (123/2242)
B) (194/2371) - (123/2365)
C) (123/2242) / (194/2177)
D) (123/2365) / (194/2371)
E) (194/2177) / (123/2242)
F) (194/2371) / (123/2365)
A newborn has cyanosis, tachypnea , and retractions of the muscles of the chest wall during inspiration. She weighs 2500 g (5 lb 8 oz). Arterial blood gas values while she breathes
room air are:
pH 7.04
Pco 2 65 mm Hg
Po 2 35 mm Hg
HCO 3- 15 mEq/L

Which of the following best describes her acid-base balance?

A) Metabolic acidosis, uncompensated


B) Metabolic alkalosis, uncompensated
C) Respiratory acidosis , uncompensated
D) Respiratory alkalosis, uncompensated
E) Respiratory acidosis and metabolic acidosis
F) Respiratory acidosis and metabolic alkalosis
A 42-year-old woman with frequent heartburn has relief of symptoms with ranitidine treatment. The therapeutic effect of ranitidine is most likely mediated by a receptor whose
activation increases which of the following in the parietal cells of the stomach?
A) Calcium
B) cAMP
C) cGMP
D) Gastrin
E) Prostaglandin E,
A physician is unable to communicate "bad news" to a patient because the patient reminds him of his older brother who always intimidated him. Which of the following terms best
describes this reaction?
A) Conversion
B) Countertransference
C) Regression
D) Splitting
E) Sublimation
A 54 -year -old man who works in a delicatessen comes to the physician because of decreased sensation of the tip of his right index finger. Six months ago, he severed this finger
while slicing meat and it was successfully reattached by prompt surgical repair. Examination of the finger shows muscle wasting . Which of the following cells is most likely blocking
reinnervation of the muscles of this patient's finger?
A) Fibrous astrocytes
B) Microglial cells
C) Oligodendrocytes
D) Satellite cells
E) Schwann cells
A previously healthy 28-year -old man comes to the physician's office because of severe abdominal pain accompanied by nausea, vomiting, and blood in the urine. Temperature is
36.7 C (98.1 F), pulse is 100/min, respirations are 20/min, and blood pressure is 140/90 mm Hg. The patient appears to be in distress and occasionally writhes in pain. Abdominal
examination shows a flat abdomen with hypoactive bowel sounds . There is mild tenderness on palpation of the right middle and right lower quadrant of the abdomen and the right
flank; there is no rebound or guarding . The testes are normal, and no hernia is palpable Which of the following is the most likely diagnosis?
A) Acute cortical necrosis
B) Acute papillary necrosis
C) Acute tubular necrosis
D) Cystitis
E) Glomerulonephritis
F) Hypernephroma
G) Interstitial nephritis
H) Nephrolithiasis
I) Pyelonephritis
A 29-year - old woman comes to the physician because of a 5-week history of fatigue and a 4-day history of heart palpitations and anxiety. She has primary hypothyroidism treated
with triiodothyronine She says that she has doubled the dose of the medication over the past week because of fatigue . Her pulse is 112/min, and blood pressure is 126/70 mm Hg
Physical examination shows a fine motor tremor of the hands. Deep tendon reflexes are brisk . Thyroid function testing is most likely to show which of the following sets of serum
concentrations?
Thyroid- stimulating Free Free
Hormone Thyroxine Triiodothyronine
A) Increased increased increased
B) Increased increased decreased
C) Increased decreased increased
D) Decreased increased increased
E) Decreased increased decreased
F) Decreased decreased increased
An 81-year -old man comes to the physician for ongoing management of hypertension of 20 years' duration. His temperature is 36.8°C (98.2°F), pulse is 80/min, respirations are
20/min, and blood pressure is 200/110 mm Hg . To reconfirm the patient's blood pressure, the physician places his index finger over the radial artery while inflating the
sphygmomanometer cuff to just above the previously measured systolic blood pressure level At this point, the radial artery becomes nonpulsatile but remains easily palpable even
as the cuff is further inflated Which of the following best explains this phenomenon?
A) Atherosclerosis
B) Connective tissue disease
C) Left ventricular hypertrophy
D) Thrombosis
E) Vasculitis
4
A previously healthy 45-year-old woman has had fever, confusion , and decreased urine production for the past 2 days. Her temperature is •
O
38.3°C (101°F). Laboratory studies show :
oe

.
Hemoglobin 9 g/dL
Hematocrit 27% 0
Leukocyte count 10,500/ mm 3

v V
Neutrophils 65% ©
Lymphocytes
Monocytes
25%
8% '6 >/ <
Eosinophils 2% Q
Reticulocyte count
Mean corpuscular volume
8%
95 pm 3
n
o©<P3
'i i
Platelet count 18,000/ mm 3
0
Prothrombin time
Partial thromboplastin time (activated)
Fibrinogen
Fibrin split products
12.8 sec (INR=1.0)
32 sec
245 mg /dL (N=200-400 mg/dL)
<10 (N=<10)
A

&
r

too
.
^ +
?
A peripheral blood smear is shown . Which of the following is the most likely cause of these findings? «

A) Aplastic anemia me
B) Disseminated intravascular coagulation
C) Fulminant liver failure
D) Thrombotic thrombocytopenic purpura
E) Vitamin B 12 (cobalamin) deficiency
A 39-year -old woman with rheumatoid arthritis comes to the physician for a follow-up examination. Treatment with multiple medications, including prednisone and methotrexate has
not been effective at slowing the progress of her condition. The most appropriate next step in pharmacotherapy is a drug that blocks the effects of which of the following cytokines?

A) Interferon gamma
B) lnterleukin-2 (IL-2)
C) IL- 4
D) IL-10
E) Tumor necrosis factor-a
A 57-year -old woman comes to the office because of a 5-week history of progressive weakness in her arms and legs. She says the weakness is most apparent when she tries to lift
dishes out of overhead cupboards and when she walks up and down flights of stairs. Muscle strength initially improves upon repetitive manual motor testing Sensation is intact in
all extremities. Which of the following mechanisms best explains this patient’s weakness?

A) Autoimmune down regulation of Ca 2+ channels of the presynaptic terminal


B) Blockade of nicotinic acetylcholine receptors at the neuromuscular junction
C) Blockade of sodium channels at the nodes of Ranvier
D) Impaired excitation-contraction coupling of the muscle
E) Increased activation of muscle K + channels
A 26-year -old woman comes to the physician 5 weeks after the birth of her first child. She worries constantly that the infant is ill and wakes him 10 to 14 times every night to make
sure he is well . She has had no crying spells or problems with appetite . She is worried that she will infect the infant with her germs and washes her own hands 30 times per day. For
2 months prior to the delivery, she worried about people breaking into her house and checked the lock on the front door three to four times every night. She is not breast-feeding
Which of the following is the most appropriate pharmacotherapy?

A) Alprazolam
B) Haloperidol
C) Lithium carbonate
D) Methylphenidate
E) Phenelzine
F) Sertraline
A 5-year-old boy is brought to the physician because of a 4-day history of red cheeks and rash over his arms and legs. He also has a 1-week history of fever. He has received all of
the scheduled childhood immunizations His vital signs are within normal limits. Physical examination shows malar erythema and a red, lacy rash over the upper and lower
extremities His hemoglobin concentration is 10 g/dL (N=11— 15). Which of the following is the most likely cause of this patient's anemia?

A) Antibody-mediated destruction of erythrocytes in the peripheral circulation


B) Destruction of erythrocytes in the peripheral circulation by an infectious agent
C) Interruption of erythrocyte production
D) Loss of erythrocytes caused by occult gastrointestinal bleeding
E) Sequestration of erythrocytes in the spleen
An 8-year - old boy is brought to the physician by his parents because they are concerned about his weight. His father has a BMI of 27 kg/m 2, and his mother has a BMI of 25 kg/m 2
The patient is at the 75th percentile for height , above the 95th percentile for weight , and above the 95th percentile for BMI Physical examination shows no other abnormalities .
Which of the following is the most likely explanation for the obesity in this patient?

A) Calorie consumption that exceeds energy expenditure


B) Decreased leptin production
C) Diet with a high fat content
D) Diet with a high glycemic index
E) Increased cortisol secretion
F) Increased insulin production
A population of vegetarians is surveyed to investigate the association between garlic/onion consumption and serum cholesterol concentration. The following results are found:
Garlic/ Onion Mean Serum
Consumption Cholesterol ( mg /dl_ )
High 159
Low 208

Which of the following tests should be used to determine whether the means are statistically different?

A) Chi-square
B) Rank correlation
C) Sign test
D) f-Test
A 60- year- old woman comes to the physician because of a 1-year history of episodes of involuntary rhythmic jerking of her right leg and foot. These episodes last approximately
3 minutes . Neurologic examination shows an increased right ankle jerk reflex A Babinski sign is present on the right . An MRI of the brain shows a 2-cm, round, enhancing lesion
within the interhemispheric fissure in the region of the central sulcus. Which of the following is the most likely diagnosis?
A) Astrocytoma
B) Lymphoma
C) Meningioma
D) Metastatic carcinoma
E) Oligodendroglioma
A 6-year- old boy who recently emigrated from Russia is brought to the physician by his parents because of unstable gait and incoordination for 2 weeks. He has had frequent pale,
bulky stools for 4 years and two episodes of bacterial pneumonia and chronic cough since the age of 1 year He is below the 3rd percentile for height and weight. Increased rhonchi
are heard over both lung fields Neurologic examination shows ataxia, absence of deep tendon reflexes , and loss of proprioception . Stool analysis shows an increased fat
concentration . Which of the following vitamins is most likely deficient in this patient?
A) Biotin
B) Niacin
C) Vitamin C
D) Vitamin D
E) Vitamin E
A 27-year - old man who is a construction worker is brought to the emergency department 30 minutes after he fell from a ladder approximately 14 feet onto a pile of boards from a
demolished house A long , sharp piece of wood penetrated his left side , immediately inferior to the middle of the body of his left 12th rib A CT scan shows that the wood sliver has
grazed the inferior pole of the left kidney and its tip is resting in the structure immediately anterior. This structure is most likely which of the following?

A) Body of the pancreas


B) Duodenum
C) Splenic flexure
D) Stomach
E) Suprarenal gland
During a clinical study. 15 patients with renal allografts volunteer to undergo treatment with Drug X . This drug is converted to a purine antagonist that interferes with the synthesis of
nucleic acids and is toxic to dividing cells. Drug X is most likely which of the following?

A) Azathioprine
B) Cyclosporine
C) Methylprednisolone
D) Rapamycin
E) Tacrolimus
A 42-year -old woman is brought to the emergency department by her husband after having a generalized tonic-clonic seizure. She tells the physician that she smelled a foul odor for
several seconds immediately before the seizure . Her husband did not perceive the odor. From which of the following regions of the brain did the seizure most likely arise?
A) Brain stem
B) Frontal lobe
C) Occipital lobe
D) Parietal lobe
E) Temporal lobe
A 35-year- old man is brought to the hospital after a diving accident. Evaluation shows complete transection of the spinal cord superior to the level of sympathetic nervous outflow.
Which of the following responses will most likely occur if he develops a systemic infection?
A) Alteration of the thermostatic set point
B) Cutaneous vasoconstriction
C) Diaphoresis following aspirin ingestion
D) Generalized involuntary shivering
E) Heat production by brown fat
A 54-year- old woman with hypertension and bilateral renal artery stenosis starts taking a nonsteroidal anti-inflammatory drug for back pain. Over the next week , her serum
creatinine concentration increases from 10 to 5.0 mg/dL. The most likely cause of this finding is the drug's ability to inhibit which of the following?
A) Inflammation in the glomerular capillaries
B) Inflammation in the renal interstitium
C) Vasoconstricting prostaglandins at the afferent arteriole
D) Vasoconstricting prostaglandins at the efferent arteriole
E) Vasodilating prostaglandins at the afferent arteriole
F) Vasodilating prostaglandins at the efferent arteriole
). A 12-year- old girl who recently immigrated to the USA from Central Africa is brought to the physician because of a 5-week history of
increasingly severe abdominal pain and vomiting blood. Her temperature is 37.8°C (100°F) , pulse is 110/min, respirations are 27/min, and
blood pressure is 112/65 mm Hg. Physical examination shows hepatosplenomegaly and abdominal ascites. Laboratory studies show :
Leukocyte count 13, 400/mm 3 (N=4500-13,500)
Segmented neutrophils 42%
Eosinophils 15%
Lymphocytes 37%
Monocytes 6%

A photomicrograph of a stool specimen is shown . Which of the following is the most appropriate pharmacotherapy for this patient?
A) Fluconazole
B) Levofloxacin
C) Metronidazole
D) Praziquantel
E) Trimethoprim-sulfamethoxazole
«
A 38 year old man who lives at sea level flies to a mountain resort in Colorado lor a skiing vacation He takes a carbonic anhydrase inhibitortabletd er arriving at the resort to help
adjusUo the altitude Two hours after drug ingestion, which of the following changes in urinary pH, bicarbonate, and volume are most likely this pa

HCOj'
pH ( mEq / L ) Volume
A) T T T
B) T i
C) T i T
D) T i 1
E) 1 T T
F) i T i
G) i i T
H) 1 i 1
A 45-year -old man with poorly controlled type 2 diabetes mellitus comes to the physician because of a 1-month history of low-grade fever He is receiving hemodialysis for end-
stage renal disease. His temperature is 37°C (98.6°F), pulse is 72/min, and blood pressure is 144/92 mm Hg. Physical examination shows a subclavian catheter inserted below the
right clavicle The lungs are clear to percussion and auscultation . Cardiac examination shows no murmurs. Two of two blood cultures grow nonhemolytic, catalase-negative, gram-
positive cocci in pairs and chains. Which of the following is the most likely causal organism?
A) Enterococcus faecalis
B) Listeria monocytogenes
C) Staphylococcus aureus
D) Staphylococcus epidermidis
E) Streptococcus pneumoniae
A 2-year- old boy is brought to the physician because of fever and cough for 2 days . He has had two episodes of Streptococcus pneumoniae bacteremia and multiple ear and sinus
infections since birth . His temperature is 38.4°C (101 1°F). Physical examination shows no tonsillar tissue. Crackles are heard in the right lower lobe of the lung Which of the
following is the most likely immunodeficiency syndrome in this patient?

A) Bruton agammaglobulinemia
B) Chediak-Higashi syndrome
C) Chronic granulomatous disease
D) DiGeorge syndrome
E) lgG2 deficiency
F) Isolated IgA deficiency
of these data, a-actinin is
myofibril from the plasma membrane in cardiac myocytes . On the basis
of the
Microelectrode injection of antibodies to a-actinin results in the detachment
micrograph of a normal cardiac myocyte shown?
most likely located at which of the following labeled sites in the electron
An 82-year -old woman has been bedridden since surgical repair of a hip fracture 6 months ago. Healing has proceeded slowly, and her physician is concerned that deep venous
thrombosis may develop . A drug that suppresses the action of which of the following factors is most likely to decrease this patients risk for thrombosis
A) Carboxylation of precursor proteins
B) Nitrous oxide
C) Prostacyclin (PGI 2)
D) Protein C
E) Proteins
F) Thrombomodulin
A 42 -year - old man comes to the physician because of numbness and tingling in both legs for 2 weeks. Sensation to vibration is decreased in the lower extremities and feet . Deep
tendon reflexes are decreased in the knees and ankles. A Romberg sign is positive . Which of the following is the most likely cause of these findings?

A) Damage to the corticospinal tracts


B) Damage to the spinothalamic tracts
C) Entrapment neuropathy
D) Radicular neuropathy affecting the anterior lumbar roots
E) Sensory neuropathy
An obese 45-year -old woman with type 2 diabetes mellitus, hypertension, dyslipidemia , and osteoarthritis of the hips and knees begins taking orlistat and starts to follow an
appropriate diet and exercise program . During the first month of therapy, this woman is at increased risk for developing which of the following conditions?

A) Anaphylaxis
B) Diarrhea
C) Increased blood pressure
D) Tachycardia
E) Tricuspid valvular insufficiency
An 84-year -old woman comes to the physician because of a 2-month history of a sore on her left hand. Physical examination shows a red , scaly, raised plaque on the dorsum of the
left middle finger. The diagnosis of squamous cell carcinoma is made The ability of this tumor to invade the basement membrane is most likely facilitated by which of the following?

A) Decreased apoptosis by tumor cells


B) Decreased concentration of laminin receptors
C) Decreased production of collagenase
D) Downregulation of E-cadherin
E) HER2/neu overexpression
A 56-year -old man with a palpable hard nodule on the prostate has an increased serum prostate-specific antigen concentration. A fine-needle biopsy specimen shows
adenocarcinoma The patient undergoes radical prostatectomy. Which of the following structures is at greatest risk for injury during removal of the prostate ?
A) Dome of the bladder
B) Obturator nerves
C) Pelvic parasympathetic nerves
D) Penile urethra
E) Ureterovesical junction
A 60-year -old man is brought to the emergency department because of the sudden onset of pain on the right side of his chest following a deep cough . An x-ray of the chest shows a
fractured rib , and a bone scan shows multiple osteoblastic lesions distributed primarily in the lumbar spine , pelvis, and ribs. A complete blood count and results of a chemistry
profiling , urinalysis, and protein electrophoresis are within the reference ranges Which of the following is the most likely diagnosis?

A) Ewing sarcoma
B) Fibrosarcoma
C) Multiple myeloma
D) Neurofibrosarcoma
E) Osteoid osteoma
F) Osteosarcoma
G) Prostate adenocarcinoma
H) Renal adenocarcinoma
I) Tuberculosis of the spine
Following a stroke , a patient is hoarse and cannot detect pinprick or cold on the left side of the face or the right side of the body Damage is most likely to have occurred in which of
the following areas of the brain on the left?

A) Base of the pons


B) Dorsal midbrain
C) Internal capsule
D) Lateral medulla
E) Postcentral gyrus
A previously healthy 60-year -old woman is admitted to the hospital following a myocardial infarction of the right ventricle. Examination shows hepatomegaly and an accumulation of
free fluid in the peritoneal space. Which of the following mechanisms is the most likely cause of the ascites and hepatomegaly?

A) Decreased capillary colloid osmotic (oncotic) pressure


B) Increased central venous pressure
C) Increased interstitial fluid colloid osmotic (oncotic) pressure
D) Increased pulmonary capillary hydrostatic pressure
E) Increased pulmonary capillary permeability
F) Lymphatic blockage
A 37-year - old woman undergoes excision of a 1-cm, painless , benign lump from the back of her neck . Seven days later, the sutured incision is erythematous and slightly warm.
There is no pain or drainage Which of the following processes best characterizes the predominant histologic appearance of the incision at this time?
A) Angiogenesis
B) Eosinophil infiltration
C) Fibrosis
D) Lymphocyte infiltration
E) Neutrophil infiltration
A 22-year-old woman comes to the office because of a 3-day history of nonproductive cough. She also has a 1- week history of fatigue, progressive shortness of breath with exertion
and while lying down, and swelling of her legs and feet . She delivered a male newborn via uncomplicated vaginal delivery 1 month ago. She has no history of major medical illness
and takes no medications. Her temperature is 37.7°C (99.8°F), pulse is 104/min , respirations are 20/min, and blood pressure is 126/80 mm Hg Bilateral basilar crackles are heard.
There is 1+ edema of the lower extremities bilaterally. Which of the following is the most likely diagnosis?
A) Amniotic fluid embolism
B) Cardiomyopathy
C) Major depressive disorder
D) Pneumonia
E) Pulmonary embolism
F) Pulmonary fibrosis
A 75-year - old woman has taken 12 over-the-counter nonsteroidal anti-inflammatory tablets daily for the past 2 weeks for mild osteoarthritis of the neck. Which of the following is
most likely to be increased in this patient?

A) Glomerular filtration rate


B) Renal blood flow
C) Plasma renin activity
D) Urine prostaglandin E,excretion
E) Urine thromboxane A 2 excretion
A 5-year- old boy who lives on a farm has had diarrhea , cramping , abdominal pain, and fever for 2 days. He drinks unpasteurized milk . An organism is isolated from the stool and
grows at 4°C. Which of the following is the most likely causal organism?

A) Bacteroides fragilis
B) Campylobacter jejuni
C) Escherichia coli
D) Helicobacter pylon
E) Salmonella typhi
F) Vibrio cholerae
G) Yersinia enterocolitica
A 45-year -old man with Li-Fraumeni syndrome agrees to participate in an investigational study of tumor suppressor gene disorders. A mutation is found in his p53 tumor suppressor
gene that converts an arginine to a proline at amino acid position 248 (R 248P). This change alters hydrogen bonding between the mutant p53 tumor suppressor gene protein and
the thymidine oxygen and adenine ring nitrogen of DNA Which of the following is the most likely result of this mutation on the transcription of genes that inhibit cell division and
contain the consensus sequence TATA (A/T) (A/T) at -30 bp 5' to the transcription start site?
A) Decreased binding of DNA polymerase
B) Decreased binding of RNA polymerase
C) Increased binding of DNA polymerase
D) Increased binding of RNA polymerase
E) Increased binding of trans factors
A 79-year - old man is brought to the emergency department by his wife 30 minutes after he lost consciousness for 30 seconds. On arrival, he is alert, but he says that he is dizzy.
There is no urinary or fecal incontinence. His pulse is 40/min and regular , and blood pressure is 92/56 mm Hg. Physical examination shows no evidence of tongue biting . The lungs
are clear to auscultation. Cardiac examination shows a variable intensity Sr He is oriented to person, place, and time. An ECG shows a third-degree atrioventricular block Which of
the following is the most appropriate next step in management?

A) Transesophageal echocardiography
B) Transthoracic echocardiography
C) Insertion of pulmonary artery catheter
D) Insertion of transvenous pacemaker
E) Cardiac catheterization with angioplasty
F) Cardiac catheterization with stent placement
A 12-year- old boy is swimming in a mountain stream He is immersed up to his neck in 60°F water for 20 minutes. Which of the following sets of physiologic changes is most likely
,

to occur in this boy?


Serum
Central Atrial
Blood Volume ADH ( vasopressin ) Natriuretic
Peptide
A) T t
B) T T i
C) T i T
D) T l I
E) i T T
F) i t l
G) i i T
H) i i i
Ten healthy human subjects are given a new oral drug and monitored for drug effect and toxicity. Blood samples are periodically taken and analyzed to define the hum
pharmacokinetics of the drug for the first time . Which of the following best describes this trial?
A) Preclinical testing
B) Phase 1
C) Phase 2
D) Phase 3
E) Phase 4
I. A 32-year- old woman comes to the physician because of a 1-day history of chest pain that radiates to her left arm She also has a 1-
week history of swelling of the left eye and a 4-day history of recurrent episodes of fever followed by shortness of breath, muscle aches,
and joint pain. Two weeks ago, she returned from a trip to the Amazon rain forest. Her temperature is 39 1°C (102 4°F), pulse is
110/min, respirations are 17/min , and blood pressure is 90/65 mm Hg. Bilateral basilar crackles are heard over the lung fields. There is
3+ edema of the lower extremities. A photomicrograph of a peripheral blood smear is shown. Which of the following is the most likely
vector for the causal organism in this patient?
A) Anopheles mosquito
B) Black fly
C) Flea
D) Ixodes tick
E) Reduviid bug
F) Sandfly
G) Tsetse fly
A) Celiac
B) Left colic
C) Right colic
D) Superior mesenteric
E) Umbilical
A 56-year - old woman comes to the physician for a follow-up examination 8 weeks after recovering from pneumococcal pneumonia . X-rays of the chest show no abnormalities
Which of the following most likely allowed this resolution to occur?

A) Formation of granulation tissue


B) Increased angiogenesis
C) Maintenance of basement membrane integrity
D) Metaplasia of mesenchymal cells to pneumocytes
E) Proliferation of fibroblasts
A 31-year - old man comes to the physician because of concerns about hair loss. He has a family history of androgenetic alopecia . Examination shows some hair thinning over the
vertex with normal scalp skin If pharmacologic treatment is elected, use of a drug to block the synthesis of which of the following steroids is most appropriate?

A) Androstenedione
B) Dihydrotestosterone
C) Estradiol
D) Estriol
E) Testosterone
A 44 - year - old man comes to the physician because of fever and mild abdominal discomfort for the past 3 days. He has a 10- year history of intravenous drug abuse. Temperature is
39.0°C (102.2°F). pulse is 108/min, and blood pressure is 150/ 94 mm Hg. The liver edge is percussed to 12 cm (4.8 in), and there is a prominent systolic pulsation of the liver.
Which of the following is the most likely diagnosis?

A) Dissecting aortic aneurysm


B) Hepatocellular carcinoma
C) Portosystemic shunt
D) Renal artery hypertension
E) Tricuspid insufficiency
A 17 - year - old boy comes to the physician because of a 1-week history of fever, fatigue , and sore throat. His temperature is 38 3°C (101°F). Physical examination shows an
erythematous oropharynx , bilateral cervical lymphadenopathy, and splenomegaly. Laboratory studies show a leukocyte count of 15, 000/mm 3 (50% typical lymphocytes and 10%
atypical lymphocytes) . A test result for heterophil antibody is positive. Twelve months after his complete recovery, which of the following is still likely to contain the virus that caused
his sore throat?
A) B lymphocyte
B) Hepatocyte
C) Macrophage
D) Renal epithelial cell
E) Segmented neutrophil
F) Sensory neuron
A 27-year - old man comes to the physician for a routine examination. He has smoked 1 pack of cigarettes daily for 14 years . When the physician initiates a discussion on smoking
cessation, the patient says, "I'd like to quit because I know it can hurt me in the long run. However, I don't think I’m ready to make such a big change now. " Which of the following is
the most appropriate next step for the physician?

A) Ask the patient to identify the pros and cons of smoking cessation
B) Detail the long-term health effects of smoking
C) Initiate pulmonary function tests
D) Prescribe bupropion
E) Recommend nicotine replacement therapy
A 39-year -old woman comes to the physician because of increasing hair growth on her face for the past year. She has not had a menstrual period during this time. A CT scan of the
pelvis shows an ovarian tumor on the left . Serum testosterone concentration is 450 ng/ dL (N=20-60 ng /dL). Which of the following laboratory values is most likely increased due to
the increased serum testosterone concentration?

A) Alkaline phosphatase
B) Follicle-stimulating hormone
C) Hemoglobin
D) High-density lipoprotein
E) Prostate-specific antigen
During a study of symptomatic proximal deep venous thrombosis , patients with this condition are randomly assigned to receive either dalteparin subcutaneously for 5 to 7 days and
then oral warfarin for 6 months , or dalteparin alone subcutaneously for 6 months No placebo controls are used. The probability of recurrent thromboembolism at 6 months is 10% in
the dalteparin group and 19% in the warfarin group Which of the following best describes this study design?
A) Case-control study
B) Case series
C) Observational cohort study
D) Open-label clinical trial
E) Open-label crossover study
A 5-year- old boy is brought to the physician because of pain in the right eye at night for the past week. There is no family history of neoplasia Examination shows strabismus and
tenderness in the eye . The left eye is normal. Examination of the retina shows the presence of a mass. The physician explains to the parents that the boy is unlikely to develop any
other neoplasms. The first mutation leading to this neoplasm most likely occurred in which of the following?
A) Germ cells
B) Retinal cells
C) Somatic cells of the child
D) Somatic cells of the parents
E) Zygote
A 78-year - old man comes to the physician because of a 3-month history of increasingly severe back pain that is worse at night and persists regardless of his activity level Physical
examination shows no abnormalities . His serum alkaline phosphatase activity is increased. An x-ray of the spine shows numerous lytic lesions along the vertebral column Which of
the following is the most likely diagnosis?

A) Chondrosarcoma
B) Ewing sarcoma
C) Metastatic carcinoma
D) Osteitis deformans (Paget disease)
E) Osteosarcoma
A 48-year - old man comes to the emergency department because he is vomiting blood. Six months ago, he had a 3-month hospital stay for severe alcohol-induced pancreatitis. His
recovery was complicated by a pseudocyst of the pancreatic tail that required surgical drainage. Examination shows splenomegaly. Hemoglobin concentration is 8.5 g/dL ; platelet
count is 56, 000/mm 3. Which of the following vessels is most likely thrombosed?
A) Gastroduodenal artery
B) Gastroduodenal vein
C) Inferior pancreaticoduodenal artery
D) Inferior pancreaticoduodenal vein
E) Left gastric artery
F) Left gastric vein
G) Splenic artery
H) Splenic vein
A 28-year - old man has recurrent pancreatitis associated with familial hypertriglyceridemia Administration of a drug with which of the following effects is most appropriate?

A) Decreasing bile acid production


B) Decreasing VLDL
C) Increasing chylomicrons
D) Increasing HDL
E) Increasing phospholipids
A newborn born at 26 weeks' gestation has respiratory distress and requires intubation and maximal oxygen support A chest x-ray shows a ground-glass appearance in both lungs
Deficient synthesis of which of the following is the most likely cause of the respiratory difficulty in this newborn?

A) Angiotensin II
B) Dipalmitoyl lecithin
C) Phosphatidylinositol 4,5-bisphosphate
D) Phosphatidylserine
E) Sphingomyelin
A 27-year-old man is brought to the emergency department 2 hours after injuring his left ankle while rock climbing. His pulse is 80/min,
respirations are 25/min, and blood pressure is 135/90 mm Hg. Physical examination shows swelling on the medial and lateral aspects of
the left ankle and severe pain on palpation. An x -ray of the left ankle is shown. This patient 's injury most likely occurred when his left foot
was forcibly moved into the extreme of which of the following positions?
A) Dorsiflexion
B) Eversion
C) Inversion
D) Plantar flexion
Two patients, a 54-year - old man (Patient X ) and a 76- year-old woman (Patient Y ), are about to receive intravenous vancomycin for severe gram -positive bacterial infections. Both
patients weigh 70 kg (154 lb) Patient X has an estimated glomerular filtration rate (GFR) of 60 mL/min , and Patient Y has a GFR of 20 mL/min. Which of the following parameters
of their drug therapy is most likely to be the same in both Patient X and Patient Y?

A) Elimination half-life
B) Loading dose
C) Serum drug concentration at 12 hours
D) Time to steady - state concentration
). A previously healthy 6-year-old boy is brought to the physician by his mother because of a 1-week history of an itchy
rash in his armpits, both hands and feet , and groin. The itching is most intense at night and keeps him awake. His
vital signs are within normal limits. Examination of the skin shows multiple erythematous papules , some with burrows
and many of which are excoriated , similar to the ones shown in the photograph. Which of the following questions to
the mother will be most helpful in establishing the diagnosis?
A) "Do you have any pets at home ?”
B) "Does anyone else in the family have an itchy rash like this?"
C) "Does your child take any medications?"
D) "Has there been any recent travel?"
E) "Have you changed soaps or detergents lately?"
A patient in the early stages of hemorrhagic shock is most likely to have which of the following?
A) Decreased ventilatory rate due to inhibition of the respiratory center
B) Flushing due to cutaneous vasodilation that occurs secondary to sweating
C) Increased renal blood flow due to sympathetic activation
D) Sweating due to parasympathetic stimulation of sweat glands
E) A weak pulse due to decreased stroke volume
A 45 -year -old man with chronic pancreatitis caused by alcohol use disorder comes to the office because of a 3-month history of diarrhea . Physical examination shows no
abnormalities. Examination of a stool specimen shows a markedly decreased amount of fecal elastase Which of the following medications is most appropriate for this patient’s
diarrhea ?
A) Ciprofloxacin
B) Lactobacillus
C) Loperamide
D) Pancrelipase
E) Somatostatin
A 48- year - old man with renal artery stenosis undergoes stent placement. Femoral access is used to place the stent. After entrance into the aorta , the guide wire should be
advanced superiorly just beyond which of the following structures to approach the right renal artery in this patient?
A) Celiac trunk
B) Internal iliac artery
C) Superior mesenteric artery
D) Testicular artery
A 13-year -old boy is brought to the physician for a well-child examination. He says that he is teased by his peers for "having breasts like a girl." He has no history of medical illness
He takes no medications. He is at the 50th percentile for height 60th percentile for weight , and 60th percentile for BMI. Physical examination shows a small, soft mound of tissue
bilaterally beneath the areola. His genital development is Tanner stage 3 . Which of the following is the most appropriate next step in management?

A) Reassurance
B) Measurement of serum growth hormone concentration
C) Measurement of serum luteinizing hormone concentration
D) Measurement of serum testosterone concentration
E) Mammography
A 62-year - old man comes to the physician's office for excision of an invasive squamous cell carcinoma on the lower lip . The physician has explained the reasons for the procedure,
the technique to be used, the possible complications of the procedure, and the identity of the physician who will be performing the procedure. Which of the following additional
information is required for informed consent?

A) Alternative treatments
B) Availability of a second opinion
C) Cost of the procedure
D) Expected recovery time
E) Medications used during the procedure
A 35-year - old man comes to the physician because of an itchy rash on his hands , waist , and feet for 2 days. He says that his wife and three teenaged children are beginning to
develop the same symptoms. He says, "We live near a wooded area and love to hike in the woods." Physical examination shows 2-mm erythematous papules in the finger webs
wrists, umbilicus, genital area , and toes. Scrapings of the lesions show mites and eggs. Which of the following is the most appropriate next step by the physician?
A) Instruct the patient to hire an exterminator to treat his home
B) Instruct the patient to use diethyltoluamide (DEET ) bug spray while hiking
C) Instruct the patient to wear a long- sleeved shirt and pants while hiking
D) Prescribe penicillin for the patient and his family
E) Prescribe permethrin for the patient and his family
A 45-year -old man with end-stage renal failure is brought to the emergency department by his mother because of his feelings of depression and apathy for 2 weeks. He has been
receiving dialysis three times weekly for 5 years , but he has missed the last three sessions because , "I've been too depressed to do anything since my wife left me 3 weeks ago. "
Bilateral crackles are heard on auscultation of the chest. Mental status examination shows confusion Serum studies are most likely to show a decrease in which of the following
concentrations?
A) Bicarbonate
B) Creatinine
C) Magnesium
D) Phosphate
E) Potassium
F) Urea nitrogen (BUN)
A 50-year - old man comes to the physician for follow -up treatment of progressive, disabling osteoarthritis during the past 6 years. The physician has just learned of a promising
experimental treatment for this condition The company that produces the new drug is offering physicians $100 for each case referred for treatment The physician believes that the
patient is a good candidate for this drug Which of the following best describes whether the physician should tell this patient that she will receive $100 for the referral?

A) No; the arrangement is contractual between the physician and the company, not the patient
B) No, because the patient may ask for the money
C) No, especially if the protocol clearly spells out that the patient is not to know
D) Yes, and offer to share the money with the patient
E) Yes ; the patient may wish to consider the money's influence on the physician's recommendation
A study is conducted to assess the prevalence of hypertension in Caucasian people compared with African American people. A total of 500 Caucasian participants (Group X ) and
500 African American participants (Group Y ) between the ages of 25 and 40 years are enrolled Blood pressure readings are taken from each participant three times over a period of
3 months. The prevalence of hypertension in Group X is 10% , and the prevalence in Group Y is 16%. Which of the following statistical tests is most appropriate to determine if this
difference is statistically significant?

A) Analysis of variance
B) Chi-square test
C) Independent f-test
D) Multiple logistic regression
E) Paired f-test
F) Pearson correlation
A 30-year - old man is brought to the emergency department 30 minutes after being stabbed in the abdomen during a robbery. His pulse is 130/min, and respirations are 28/ min. His
mean arterial pressure is 50 mm Hg , and pulse pressure is 5 mm Hg to 10 mm Hg. A diagnosis of rupture of the aorta is made Which of the following is expected to occur as a
compensatory response to this rupture?

A) Decreased serum ADH (vasopressin) concentration


B) Decreased serum urea nitrogen plasma creatinine ratio
C) Decreased total peripheral resistance
D) Increased capillary hydrostatic pressure
E) Increased carotid sinus nerve firing rate
F) Increased serum angiotensin II concentration
A 35-year - old man is admitted to the hospital because he has been vomiting blood . He has a 15-year history of alcohol dependence and biopsy -proven cirrhosis. Examination
shows ascites, prominent periumbilical veins, and hemorrhoids. Over the next 7 days, his level of consciousness decreases and he develops a flapping tremor of the hands. The
pathogenesis of the neurologic symptoms is most likely directly related to which of the following?

A) Absorption of nitrogenous products from the gastrointestinal tract


B) Hypoxia caused by decreased respiratory effort resulting from ascites
C) Shunting of systemic blood into the portal system
D) Systemic hypotension caused by blood loss
E) Thrombosis of hepatic veins
A 23-year- old woman develops multiple red, papulovesicular lesions following exposure to poison ivy Which of the following is the most likely mechanism underlying the
development of these lesions?

A) Allergen-mediated vasoconstriction , leading to ischemic tissue injury


B) Binding of antigen to IgE on the surface of mast cells, leading to mast cell degranulation
C) Deposition of antigen- antibody complexes within postcapillary venules, leading to activation of complement
D) Phagocytosis of antigen by neutrophils , leading to oxidant-mediated tissue damage
E) Processing of antigen by Langerhans cells, leading to activation of CD 4+ T cells
An 18-year -old woman develops sepsis after an abortion . Within 24 hours she becomes dyspneic, oliguric, and develops petechiae , ecchymoses, and bleeding from venipuncture
sites . Which of the following is the laboratory finding most consistent with this disease process?
A) Decreased bleeding time
B) Decreased plasma fibrinogen concentration
C) Increased antineutrophil antibody titers
D) Increased plasma factor VIII (antihemophilic factor ) concentration
E) Increased plasma hemolytic complement activity
A 21-year - old man is brought to the emergency department 1 hour after the acute onset of left - sided chest pain and shortness of breath . The symptoms began while he was playing
basketball with friends. He is 178 cm (5 ft 10 in) tall and weighs 63 kg (138 lb); BMI is 20 kg/m 2 His pulse is 120/min, respirations are 30/min, and blood pressure is 124/ 86 mm Hg
Breath sounds are decreased on the left. There is hyperresonance to percussion of the left hemithorax . A chest x-ray shows a sharp line adjacent to a radiolucent region without
lung markings on the left side Which of the following is the strongest predisposing risk factor for this patient's condition?

A) Collision with another player


B) Family history of asthma
C) Gender and body habitus
D) History of tobacco use
E) Playing basketball
A 13-year- old girl is brought to the emergency department 15 minutes after the sudden onset of shortness of breath. She has a 9-year history of episodes of soft tissue swelling,
particularly involving her face and extremities, but without urticaria or erythema. Some of these episodes have been accompanied by acute abdominal pain with diarrhea . She has
no history of severe or frequent infections. Her temperature is 37°C (98.6°F), pulse is 81/min, respirations are 20/min and labored with marked stridor, and blood pressure is 95/65
mm Hg . Physical examination shows marked facial edema. Treatment with danazol and stanozolol resolves her symptoms. The most likely cause of her condition is a deficiency of
which of the following substances?

A) C1 inhibitor
B) CD 40 ligand
C) Factor VIII (antihemophilic factor)
D) IgA
E) NADPH oxidase
F) Plasminogen activator
A 50-year - old man with chronic gastritis is diagnosed with a marginal zone lymphoma of the stomach. Which of the following organisms is most likely associated with the
development of this lymphoma?

A) Cryptosporidium parvum
B) Cyclospora cayetanensis
C) Epstein-Barr virus
D) Helicobacter pylori
E) HIV-1
F) HTLV-1
A 52-year - old man with stable angina pectoris begins treatment with a medication that decreases cardiac preload and increases coronary artery blood flow. This patient is at greatest
risk for which of the following adverse effects of this medication?

A) Bradycardia
B) Constipation
C) Headache
D) Hearing loss
E) Nausea
F) Visual disturbance
A 62-year - old man comes to the physician for a follow -up examination . He has hypertensive nephropathy and chronic renal insufficiency. Current medications include clonidine and
calcium acetate . His blood pressure is 145/92 mm Hg. Laboratory studies show a serum creatinine concentration of 3.5 mg/dL and serum calcium concentration of 7.8 mg /dL.
Which of the following additional sets of serum concentrations is most likely in this patient?
Inorganic Parathyroid
Phosphorus Hormone Calcitriol
A) Increased increased decreased
B) Increased decreased increased
C) Increased decreased decreased
D) Decreased increased increased
E) Decreased increased decreased
A 56-year- old man develops brief episodes of ventricular tachycardia 14 hours after having an acute myocardial infarction . The arrhythmia will be treated with a loading dose of
lidocaine followed by a continuous infusion of the same drug over the next 24 hours at a rate to maintain a therapeutic steady-state drug concentration (Css) in plasma. To maintain
a Css of lidocaine, the appropriate rate of infusion of the drug is best determined by which of the following calculations? (CL=drug clearance, Vd=volume of distribution, t1/2=half-life)

A) CL x Css
B) CL / Css
C) CL x V/2
D) CL / V/2
E) Vd x Css
F) Vd / Css
G) Vdxt1/2
H) Vd / V/2
A 3-month-old girl is brought to the physician by her mother for a well-child examination. Her mother says that the infant can lift her head when lying prone. The patient is eating well
and recently started sleeping about 6 uninterrupted hours through the night . The mother adds that the baby does not yet smile in response to human faces and has not started to
coo or gurgle with attention The patient is at the 25th percentile for length and weight and 10th percentile for head circumference. Physical examination shows no other
abnormalities Which of the following best describes this infant’s development?
Motor Social Verbal and
Cognitive
A) Normal normal normal
B) Normal normal delayed
C) Normal delayed normal
D) Normal delayed delayed
E) Delayed normal normal
F) Delayed normal delayed
G) Delayed delayed normal
H) Delayed delayed delayed
A 29-year- old woman has an inflammatory disease involving her joints, kidneys, serosal surfaces, anterior chamber of the eye, and choroid plexus. Which of the following is the
most likely mechanism of this disorder?
A) Antibasement membrane antibody-dependent cell-mediated cytotoxicity
B) Antibasement membrane antibody-dependent complement-mediated cytotoxicity
C) Cell-mediated cytotoxicity
D) Immune complex-mediated cytotoxicity
E) Release of vasoactive compounds from IgE- stimulated basophils
F) Systemic release of interleukin-1 (IL-1) and tumor necrosis factor
A 72-year -old man with multiple myeloma agrees to participate in an investigational chemotherapy clinical trial Prior to enrollment in the study, peripheral blood B and T
lymphocytes are isolated and undergo Southern blot analysis of lymphoid cell-specific genes. Using a single B-lymphocyte J-region probe , analysis of the B-lymphocyte DNA shows
a 1 5-kb band. Analysis of the T-lymphocyte DNA using the same J-region probe shows a 6-kb band The 6-kb band most likely signifies which of the following?
,

A) Constant region gene rearrangement


B) D to J gene rearrangement
C) V to D gene rearrangement
D) V to J gene rearrangement
E) Unrearranged immunoglobulin gene
A 38-year - old man with Down syndrome is brought to the physician by his nephew for a follow-up examination. He had lived with his parents until their death 2 years ago, but he
now lives with his sister and her children He receives Social Security insurance benefits from his deceased father's account , and this is currently the family' s only income He has
missed his last four scheduled visits He is unkempt and malodorous, and he appears anxious and hypervigilant. He startles when his nephew speaks, and he shrinks away when
the physician reaches to pat his shoulder. The patient used to be pleasant and relaxed He has had a 6.4-kg ( 14-lb) weight loss since his last appointment 3 months ago. When the
physician asks to interview the patient alone, the nephew refuses to leave the room. Which of the following is the most appropriate initial action by the physician?

A) Ask the patient with the nephew present if he feels safe at home
B) Ask the patient's nephew if he often loses his temper around the patient
C) Contact adult protective services
D) Determine if the patient's nutritional needs are being met
E) Meet with the patient's family and advise them on how to care for someone with Down syndrome
A 53-year - old woman shows evidence of adrenal failure. On CT scan , both adrenal glands appear normal in configuration but decreased in size. Which of the following is the most
likely diagnosis?

A) Autoimmune adrenalitis
B) Disseminated coccidioidomycosis
C) Metastatic carcinoma from the lung
D) Retroperitoneal idiopathic fibrosis
E) Tuberculosis
A 45-year - old woman comes to the physician for a follow- up examination 8 weeks after beginning tamoxifen therapy for estrogen- and progesterone-positive invasive ductal
carcinoma of the breast Her 50-year- old sister also has hormone- sensitive breast cancer treated with tamoxifen Physical examination shows no abnormalities Serum studies
show decreased concentrations of endoxifen, the active metabolite of the prodrug tamoxifen . Genetic analysis shows the homozygous presence of cytochrome P450 2D6 * 4 alleles.
Which of the following best represents the likelihood that this patient's sister has the same alleles?
A) 0%
B) 25%
C) 50%
D) 75%
E) 100%
S. A 22-year - old man comes to the physician because of infertility. The photomicrograph shows tissue obtained on biopsy of the scrotal
testes . Which of the following is the most likely genetic abnormality?
A) 45,X/ 46, XY
B) 46,XX
C) 46,XY
D) 47 ,XXX
E) 47,XXY
A 42-year - old man is brought to the emergency department because he says he feels as if he is "frozen in ice . " Examination shows muscle rigidity. Toxicology screen shows
evidence of exposure to MPTP (N-methyl- 4-phenyl-1,2,3,6-tetrahydropyridine). Which of the following neuronal cell bodies are primarily affected?

A) Cholinergic neurons in the nucleus basalis of Meynert


B) Dopaminergic neurons in the neostriatum
C) Dopaminergic neurons in the substantia nigra
D) Serotonergic neurons in the dorsal raphe nuclei
E) Serotonergic neurons in the locus ceruleus
A 3-day-old full-term male newborn is brought to the physician by his mother because of a 24-hour history of yellow skin. He appears healthy. Physical examination shows mild
jaundice. Laboratory studies show a hemoglobin concentration of 17 g/dL and a total serum bilirubin concentration of 10 mg/dL, with an indirect component of 8 mg/dL. The jaundice
resolves 5 days later. A deficiency of which of the following is the most likely cause of the jaundice in this patient?

A) Erythrocyte glucose 6-phosphate dehydrogenase activity


B) Erythrocyte pyruvate kinase activity
C) Glutathione synthetase activity
D) Hepatic excretion of bilirubin
E) Hepatic UDP-glucuronosyltransferase activity
A 26-year - old nulligravid woman comes to the physician because she has been unable to conceive for the past year She also has had pain during sexual intercourse. Menses occur
at irregular 2- to 3-month intervals . Her last menstrual period was 8 months ago Physical examination shows no abnormalities. Serum studies show decreased concentrations of
estrogen , follicle- stimulating hormone (FSH), and luteinizing hormone (LH ). Dysfunction of which of the following is the most likely cause of her infertility and dyspareunia?

A) Adrenal gland
B) Hypothalamus
C) Ovarian FSH receptors
D) Ovarian LH receptors
E) Vaginal estrogen receptors
An 86-year -old man who lives in a skilled nursing care facility is brought to the physician because of a 3-week history of chronic nonproductive cough . He says that he is tired in the
mornings because the cough interrupts his sleep Physical examination shows no abnormalities except for the cough . A chest x-ray shows no abnormalities In addition to further
evaluation to determine the cause of his cough, which of the following drugs is most appropriate to treat this patient while avoiding the adverse effect of constipation?

A) Codeine
B) Dextromethorphan
C) Diphenhydramine
D) Morphine
E) Tiotropium
Which of the following drug effects is the most common reason for noncompliance with cyclic antidepressant therapy?
A) Antiadrenergic
B) Anticholinergic
C) Antihistaminic
D) Cholinomimetic
E) Sympathomimetic
A 65-year-old man comes to the physician for a follow-up examination. He has a 15-year history of poorly controlled type 2 diabetes mellitus, resulting in multiple peripheral
neuropathies . Neurologic examination shows wasting of the interosseous muscles of the left hand and inability to abduct the fingers of this hand; plantar flexion of the right foot is
absent. The function of which of the following pairs of nerves is most likely impaired in this patient?

A) Median and common fibular (peroneal)


B) Median and tibial
C) Radial and common fibular (peroneal)
D) Radial and tibial
E) Ulnar and common fibular (peroneal)
F) Ulnar and tibial
A 72-year - old man is given ketorolac for pain control after an abdominal operation. He has a 20-year history of hypertension and type 2 diabetes mellitus . He is at greatest risk for
which of the following drug-induced adverse effects?

A) Acute renal failure


B) Addiction to the pain medication
C) Disorientation
D) Hypotension
E) Sepsis
). A 62-year - old man is being evaluated for rectal bleeding. An x-ray of the gastrointestinal tract with contrast material is shown.
Which of the following is the most likely explanation for the feathery appearance in the portion of the gastrointestinal tract
indicated by X when compared with the portion indicated by Y ?
) A) Absence of circular muscle
B) Fewer villi
C) Greater bowel motility
D) Greater mucosal surface area
) E) Less circular and longitudinal smooth muscle
A 35-year-old man comes to the physician because of a 3-year history of an enlarging nose, coarsening of his facial features, muscle weakness, and increased hand and foot size.
Physical examination shows a large fleshy nose and prognathism. Serum studies show an increased insulin-like growth factor- 1 concentration. An MRI of the brain shows a pituitary
adenoma. Morphologic analysis of a biopsy specimen of the tumor shows a densely granulated somatotroph adenoma. Further studies show that the Gas subunit of G proteins in
the tumor lack GTPase activity. The tumor cells in this patient most likely have an increased activity of which of the following enzymes?

A) Adenylyl cyclase
B) Guanylyl cyclase
C) Janus kinase
D) Phospholipase C
E) Tyrosine kinase
A 52-year - old man comes to the physician because a lump in his inguinal region, which had been present for 4 months, has recently enlarged and become painful. It is determined
that a direct inguinal hernia is present , and a segment of gut has pushed through the superficial inguinal ring. Which of the following will be impaired if the herniated gut compresses
the nerve that lies on the external surface of the spermatic cord at the superficial inguinal ring?

A) Motor innervation to the bulbocavernosus muscle


B) Motor innervation to the lower portion of the rectus abdominis muscle
C) Sensation from the anterior surface of the scrotum
D) Sensation from the dorsal surface of the penis
Clostridium perfringens-a toxin affects cells and facilitates the spread of gas gangrene by which of the following mechanisms?

A) ADP-ribosylation of elongation factor-2


B) Formation of pores
C) Preventing release of inhibitory neurotransmitters
D) Proteolytic cleavage
E) Splitting lecithin to phosphorylcholine and diglyceride
He has a tremor of his head and hands that is worse when he
A 40-year-old woman comes to the physician because of a 6-month history of dryness of her mouth and itching and burning of her eyes. Physical examination shows inflammation
of both corneas and sclerae. enlarged salivary glands , and a paucity of saliva . Serum antinuclear antibody assay is positive. She is told that she is at risk for developing a
lymphoma . Which of the following is the most likely mode of inheritance?

A) Autosomal dominant
B) Autosomal recessive
C) X-linked dominant
D) X -linked recessive
E) Multifactorial
An 18-month-old boy is admitted to the hospital because of generalized tonic-clonic seizures. He has a history of mild developmental delay. There is no family history of seizure
disorder or major medical illnesses. He is at the 25th percentile for length, the 25th percentile for weight , and the 25th percentile for head circumference. Physical examination
shows six hypopigmented macules scattered on the torso and a raised , flesh-colored lesion on the back . An MRI of the brain shows multiple lesions in the cerebral cortex . Which of
the following is the most likely diagnosis?
A) Ataxia -telangiectasia
B) Incontinentia pigmenti
C) Neurofibromatosis 1
D) Sturge-Weber syndrome
E) Tuberous sclerosis complex
A 62-year - old woman comes to the physician 3 days after discovering a painless mass in her left breast during self-examination. She has not had any weight loss or other recent
symptoms. Mammography 18 months ago showed no abnormalities. Menopause occurred at the age of 50 years. She takes no medications. She has no family history of breast
cancer. Physical examination shows a 2-cm, firm, nontender mass in the upper outer quadrant of the left breast . There is no axillary lymphadenopathy or nipple discharge.
Mammography shows an extremely radiodense mass with irregular margins; it contains clustered irregular microcalcifications. Which of the following is the most likely diagnosis?

A) Breast abscess
B) Carcinoma of the breast
C) Fibroadenoma
D) Fibrocystic changes of the breast
E) Intraductal papilloma
A 65-year-old man comes to the emergency department because of a 1-day history of shortness of breath . He has hypertension and type 2 diabetes mellitus. There is no history of
heart disease. Current medications include hydrochlorothiazide, ramipril, and metformin . He does not smoke. His temperature is 37°C (98.6°F), pulse is 110/ min , respirations are
28/min, and blood pressure is 152/92 mm Hg. Pulse oximetry on room air shows an oxygen saturation of 90%. Cardiac examination shows an S 3. A grade 2/6 systolic murmur is
heard over the lower left sternal border radiating to the axilla . The point of maximal impulse is diffuse and most prominent in the axillary line. Which of the following findings is most
likely on pulmonary auscultation?

A) Crackles
B) Egophony
C) Unilateral decreased breath sounds
D) Wheezing
E) Whispered pectoriloquy
Thirty minutes after taking 2 aspirin tablets for a tension headache, a 17- year -old girl has difficulty breathing . An adverse reaction to the aspirin is suspected. In the future , she should
take which of the following drugs to treat her headaches?
A) Acetaminophen
B) Ibuprofen
C) Ketorolac
D) Naproxen
E) Sumatriptan
A 38-year -old man is admitted to the hospital after sustaining multiple injuries in a farm implement accident . The day after admission, his plasma fibrinogen concentration is
increased Which of the following is the most likely cause of this finding?
A) Acute-phase response
B) Anaphylaxis
C) Disseminated intravascular coagulation
D) Hemoconcentration
E) Histamine triple response
Nicotinic acid acts at which of the following labeled sites in the diagram of lipid metabolism?

Hepatocyles
"

Blood \
Gut Ac - CoA
i
LDL
^T LDL m
a
o
HMG CoA-
IB LDL zr
Cholesterol <D
L

A
I
ile acids c
\ ? at
o

O A) OB) O C) O D) O E)
A 31-year - old woman in the second trimester of pregnancy is brought to the emergency department because of vaginal bleeding. Blood pressure is 140/ 95 mm Hg Ultrasound
examination shows a conceptus with patches of tissue containing vesicular structures but no apparent embryo. The tissue is surgically removed and genetic analysis shows a
.
69 XXY karyotype The presence of which of the following would indicate that an embryo had been formed within this conceptus?

A) Cytotrophoblast
B) Hematopoietic stem cells
C) Primary stem villus
D) Syncytiotrophoblast
E) Trophoblastic lacunae
A new virus has been isolated that causes encephalitis. The viral particle contains an RNA-dependent DNA polymerase . Which of the following types of RNA is most likely to be the
genome of this virus?

A) Double-stranded circular
B) Double-stranded linear
C) Single- stranded negative-sense
D) Single-stranded positive-sense
A 53-year- old woman with a long history of fibromuscular dysplasia involving the left renal artery develops hypertension . Her left kidney is 7.4 cm (N = 10 cm). She is being
evaluated for a revascularization procedure; a left kidney biopsy is performed Which of the following histologic findings is most likely?

A) Deposition of fibrinoid deposits within interlobular arteries


B) Diffuse infiltrate of neutrophils within tubules and interstitium
C) Diffuse mesangial hypercellularity
D) Severe atherosclerosis involving interlobular arteries and hyaline arteriosclerosis involving arterioles
E) Tubular atrophy
A 76-year - old man with a 1-month history of a pulsatile abdominal mass is diagnosed with an abdominal aortic aneurysm. CT scans of the abdomen with contrast also showed an
incidental finding of a horseshoe kidney. In this setting , the surgical approach to repair of the aneurysm in this patient is most likely to be further complicated by the presence of
which of the following?
A) Abnormal origin of the superior mesenteric artery
B) Anomalous origins of multiple renal arteries to each kidney
C) Friability of vascular tissue as a result of collagen synthesis abnormalities
D) Multiple ureters with abnormal courses
E) Tumors with malignant degeneration
A 73-year - old woman dies 7 years after the onset of progressive memory loss and depression . She had severe intellectual
deterioration at the time of death . A coronal section of the brain at autopsy is shown . Histologic sections showed numerous
neuritic plaques in the cerebral cortex and numerous neurofibrillary tangles in cortical neurons. Which of the following is the most
likely cause of the ventricular enlargement?
A) Communicating hydrocephalus
) B) Hydrocephalus ex vacuo
) C) Noncommunicating hydrocephalus
) D) Normal pressure hydrocephalus
E) Oversecretion hydrocephalus
A 24 -year - old man who is comatose is admitted to the hospital because of a drug overdose and pneumonia. On admission , he is intubated and mechanically ventilated While
receiving a tidal volume of 500 mL, the patient's end- expiratory pressure is + 5 cm H 20, and his end-inspiratory airway pressure is +25 cm Hp An esophageal balloon is inserted
and an end-inspiratory pleural pressure of +20 cm H 20 is measured. Which of the following best represents this patient's respiratory system compliance?
A) 0.01 cm Hp/mL
B) 0.04 cm Hp /mL
C) 0.05 cm Hp/mL
D) 20 mL/cm H 20
E) 25 mL/cm H20
F) 100 mL/cm Hp
An 1814- g (4-lb) male newborn is delivered in the hospital at 39 weeks' gestation to a 40-year -old woman, gravida 4, para 4 via an uncomplicated vaginal delivery Shortly after
delivery, he develops respiratory distress and dies 8 hours later At autopsy, examination of the pancreas shows congenital absence of the pancreatic islets The most likely cause
of this congenital abnormality is a defect in the initial differentiation of islet cells from which of the following precursor cells?

A) Endodermal cells
B) Epithelial cells in the yolk sac wall
C) Neural crest cells
D) Paraxial mesodermal cells
E) Splanchnic mesodermal cells
A 36-year - old woman comes to the physician because of a 4-month history of chronic fatigue. Physical examination shows a macular rash over the face. Serum studies show the
presence of antibodies against nucleosomes but not against histones. Urinalysis shows blood and 3+ protein. A biopsy specimen of renal tissue is consistent with lupus nephritis.
Treatment with prednisone is begun. This drug acts by binding to its receptor in which of the following sites?

A) Cytosol with translocation into the nucleus


B) Lysosome
C) Outer mitochondrial membrane
D) Outer nuclear membrane with translocation into the inner nuclear membrane
E) Peroxisome
F) Plasma membrane with translocation into the endosome
A 10-year - old boy has bruised easily since swallowing some of his grandfather's pills 2 days ago. Aspirin and dicumarol are found in the medicine cabinet. Which of the following
laboratory findings would suggest that he took dicumarol rather than aspirin?

A) Decreased plasma fibrinogen concentration


B) Decreased partial thromboplastin time
C) Decreased platelet count
D) Normal clotting time
E) Prolonged prothrombin time
A 62-year - old man with dyslipidemia is brought to the emergency department by his wife because of a 6-hour history of severe muscle aches , generalized weakness, and brown
urine His wife says that he recently started a new lipid-lowering drug that was added to his medication regimen including simvastatin, losartan, chlorthalidone, and atenolol. His
pulse is 90/min and regular, and blood pressure is 96/72 mm Hg Physical examination shows no other abnormalities. His serum creatinine concentration is 10.2 mg/dL. Urinalysis
shows 4+ blood but no RBCs. Which of the following medications was most likely added to this patient’s drug regimen?
A) Cholestyramine
B) Colesevelam
C) Ezetimibe
D) Gemfibrozil
E) Niacin
A 38-year-old single woman with a history of chronic headaches comes to the physician for a follow-up examination. She has three sons with sickle cell disease , for which they
have required frequent hospitalizations. She tells the physician that she used to feel angry and abandoned by her family, as they have provided little to no support in helping her
with her sons. She states that she recently started taking kickboxing lessons, and now she feels happier and more relaxed. She adds that her headaches also have improved. This
patient is most likely using which of the following types of coping mechanisms?

A) Displacement
B) Projection
C) Rationalization
D) Reaction formation
E) Suppression
Parenterally administered cholera vaccines consisting of killed whole bacterial cells have very limited efficacy. Which of the following is the primary reason for this failure?

A) Ability of Vibrio cholerae to produce multiple antigenic types of enterotoxins


B) Antigenic variation in adhesive pili
C) Inability of the vaccine to elicit secretory antibody at the epithelial surface
D) Toxicity of the vaccine preparation
A 17-year - old primigravid woman at 16 weeks' gestation has a mucopurulent vaginal discharge. Culture of the discharge grows Chlamydia trachomatis. Which of the following is the
most appropriate pharmacotherapy?
A) Ampicillin
B) Azithromycin
C) Chloramphenicol
D) Doxycycline
E) Trimethoprim-sulfamethoxazole
A 6-month-old boy has recently been diagnosed with developmental delay. He has had difficulty feeding and recurrent vomiting since early infancy. He was found to have
hepatosplenomegaly and lymphadenopathy during a routine examination at age 3 months . Examination of a biopsy specimen of bone marrow shows numerous foam cells. A defect
in which of the following metabolic pathways is the most likely cause of this disorder?
A) Cholesterol-ester degradation
B) Chylomicron transport
C) Lipoprotein degradation
D) Sphingomyelin degradation
E) VLDL synthesis
During the first week of life, a male newborn has vomiting, severe dehydration, hyponatremia, and salt wasting . Serum 17-hydroxyprogesterone concentration is increased The
most likely cause of his disorder is a defect in which of the following enzymes?
A) Desmolase
B) 17a-Hydroxylase
C) 21-Hydroxylase
D) 1ip-Hydroxylase
E) 3P-Hydroxysteroid dehydrogenase
A 52-year -old man with a history of alcoholic cirrhosis is brought to the office by his wife because of a 4-hour history of rectal bleeding He has a history of alcohol use disorder and
hemorrhoids . His pulse is 100/min, and blood pressure is 110/80 mm Hg. Physical examination shows pallor. The varicosities in this patient are most likely located in direct
tributaries of which of the following veins?

A) Inferior mesenteric
B) Inferior rectal
C) Superior mesenteric
D) Superior rectal

Das könnte Ihnen auch gefallen